You are on page 1of 83

ACE

Engineering Publications
(A Sister Concern of ACE Engineering Academy, Hyderabad)

Hyderabad  Delhi  Bhopal  Pune  Bhubaneswar  Bengaluru  Lucknow  Patna  Chennai  Vijayawada  Visakhapatnam  Tirupati  Kolkata  Ahmedabad

Numerical Ability
(Logical reasoning and Analytical ability)

&
Verbal Ability
(Practice Questions Booklet)

ACE is the leading institute for coaching in ESE, GATE & PSUs
Address: Sree Sindhi Guru Sangat Sabha Association, # 4-1-1236/1/A, King Koti, Abids, Hyderabad-500001.
Ph: 040-23234418 / 19 / 20 / 21, 040 - 24750437

15 All India 1st Ranks in ESE


57 All India 1st Ranks in GATE
Copyright © ACE Engineering Publications

All rights reserved.

No part of this publication may be reproduced, stored in a retrieval system, or transmitted,


in any form or by any means, electronic, mechanical, photocopying, digital, recording or
otherwise, without the prior permission of the publishers.

Published at:

ACE Engineering Publications


Sree Sindhi Guru Sangat Sabha Association,
# 4-1-1236/1/A, King Koti, Abids,
Hyderabad – 500001, Telangana, India.
Phones: 040- 40044403 / 040-23234419
Website: aceengineeringpublications.com
Email: aceenggpublications@aceenggacademy.com
hyderabad@aceenggacademy.com

Authors:
Subject experts of ACE Engineering Academy, Hyderabad

While every effort has been made to avoid any mistake or omission, the publishers do not owe any
responsibility for any damage or loss to any person on account of error or omission in this publication.
Mistakes if any may be brought to the notice of the publishers, for further corrections in
forthcoming editions, to the following Email-id.
Email : aceenggpublications@aceenggacademy.com
Foreword

Dear Students,

The Practice Questions for Numerical & Verbal Ability is thoroughly revised this year to meet the
requirements of all categories of students.

After attending the classes and understanding the typical questions explained by the professors,
the student will be in a position to solve these questions in work book. However, for verifying the
correctness, key is also given wherever required. The student is advised to try to get the answer for
the questions by self analysis and not to refer to the key.

Thanks to all the Professors who extended their valuable services in the preparation of this Booklet.
It is believed that this volume is also a valuable aid to the students appearing not only for GATE
but also other for competitive exams like ISRO, State service Commissions, Campus placements
and other PSUs.

With best wishes to all the Students

Y.V. Gopala Krishna Murthy,


M Tech. MIE,
Chairman & Managing Director,
ACE Engineering Academy,
ACE Engineering Publications.
SYLLABUS

Numerical Ability

GATE: Numerical computation, numerical estimation, numerical reasoning and data


interpretation

ESE: Engineering Aptitude covering Logical reasoning and Analytical ability

Verbal Ability
GATE: English grammar, sentence completion, verbal analogies, word groups, instructions,
critical reasoning and verbal deduction.
CONTENTS

NUMERICAL ABILITY VERBAL ABILITY

1. Numerical Estimations 1 1. Grammar 79


1.1 The Noun ...................................................................... 80
2. Numerical Reasoning 6 1.2 The Pronoun ............................................................... 81
2.1 Number, Time Sequence Test, Ranking, & 1.3 The Verb ....................................................................... 82
6
Comparison........................................................................
1.4 Tense/ Voice Chart ................................................... 83
2.2 Directions and Distances............................................. 9
1.5 Present Tense ............................................................. 86
2.3 Arithmetical Reasoning............................................... 11
1.6 Present Perfect Tense ............................................. 87
2.4 Blood Relations............................................................... 14
1.7 Past Tense .................................................................... 88
2.5 Cubes and Dice ............................................................... 17
1.8 Simple Past Tense ..................................................... 88
2.6 Coding and Decoding Tests........................................ 20
1.9 Past Perfect ................................................................. 88
2.7 Inserting the Missing Character............................... 23
1.10 Future Tense .............................................................. 89
2.8 Analytical Reasoning..................................................... 27
1.11 Subject-verb Agreement ....................................... 90
2.9 Analytical Figures........................................................... 31
1.12 The Adjective ............................................................. 92
2.10 Logical Reasoning........................................................ 33
1.13 The Adverb ................................................................. 96
2.11 Verbal Test of Reasoning.......................................... 36
1.14 The Preposition ........................................................ 97
1.15 The Conjunction ....................................................... 102
3. Numerical Computation 42
3.1 Number system............................................................... 42 107
2. Pairs That Snare
3.2 Ratio, Proportion and Variation............................... 44 127
3. Analogy
3.3 Partnership........................................................................ 46 130
4. Sentence Completion
3.4 Averages.............................................................................. 47 134
5. Critical Reasoning
3.5 Problem on Ages............................................................ 49 Key & Detailed Explanations 140
3.6 Time and Work................................................................ 50
3.7 Pipes and Cisterns.......................................................... 52
3.8 Time, Speed and Distance........................................... 53
3.9 Mixture and Alligation.................................................. 56
3.10 Percentage....................................................................... 57
3.11 Profit, Loss and Discount......................................... 60
3.12 Simple and Compound Interest............................. 61

4. Data Interpretation 63
4.1 Tabulation ........................................................................ 63
4.2 Bar Charts ........................................................................ 64
4.3 Pie Charts ......................................................................... 66
4.4 Line Graphs / X - Y Charts ........................................ 67
n

NUMERICAL
ABILITY
Chapter 1 Numerical Estimation
Logical Puzzles

01. The digit in the units place of the number (17)1999 + (11)1999 – (7)1999 is ____.
(a) 7 (b) 4 (c) 1 (d) 3

02. If the unit’s digit in the product (47x x 729 x 345 x 343) is 5, what is the maximum number of values that x may
take?
(a) 9 (b) 3 (c) 7 (d) 5

03. Find the last digit of the product 2222 x 3333 x 4444 x 5555?
(a) 3 (b) 2 (c) 1 (d) 0

04. What will be the last digit of the multiplication 3153 x 7162?
(a) 5 (b) 9 (c) 7 (d) 6

05. When 4157 is multiplied with 7113 what is the digit in the least significant place?
(a) 7 (b) 4 (c) 8 (d) 2

06. The last digit of (2171)7 + (2172)9 + (2173)11 + (2174)13 is (CE – 2017)
(a) 2 (b) 4 (c)6 (d) 8

07. The numeral in the units position of 211870 + 146127 x 3424 is ____. (EE-2016)

08. The number of zeros at the end of the product 16 x 22 x 15 x 50 x 65 x 115 x 18 x 90 is


(a) 5 (b) 6 (c) 12 (d) 7

09. There are eight bags of rice looking alike, seven of which have equal weight and one is slightly heavier. The
weighing balance is of unlimited capacity. Using this balance, the minimum number of weighings required to
identify the heavier bag is (EC/EE/INST-2012)(2M)
(a) 2 (b) 3 (c) 4 (d) 8

10. Raju has 14 currency notes in his pocket consisting of only₹.20 notes and ₹10 notes. The total money value of the
notes is 230. The number of ₹10 notes that Raju has is (EC/EE/INST-2012)(2M)
(a) 5 (b) 6 (c) 9 (d) 10

11. My friend Asha was throwing a very grand party and wanted to borrow from me 100 wine glasses. I decided to
send them through my boy servant Harish. Just to give an incentive to Harish to deliver the glasses intact I offered
him 3 paise for every glass delivered safely and threatened to forefeit 9 paise for every glass he broke. On settle-
ment Harish received ₹.2.40 from me. How many glasses did Harish break?
(a) 4 (b) 5 (c) 6 (d) 10
3 Numerical Estimation

12. A window is made up of a square portion and an equilateral triangle portion above it. The base of the triangular
portion coincides with the upper side of the square. If the perimeter of the window is 6m, the area of the window
in m2 is ___ (ME – 2016) (1M)
(a) 1.43 (b) 2.06 (c) 2.68 (d) 2.88

13. It is a small town railway station and there are 25 stations on that line. At each of the 25 stations the passengers
can get tickets for any of the other 24 stations. How many different kinds of tickets do you think the booking clerk
has to keep.
(a) 24 (b) 240 (c) 250 (d) 600

14. Mammu wears socks of two different colours – white and brown. She keeps them all in the same drawer in a state
of complete disorder. She has altogether 20 white socks and 20 brown socks in the drawer. Supposing she has to
take out the socks in the dark, how many must she take out to be sure that she has a matching pair?
(a) 2 (b) 3 (c) 4 (d) None of these.

15. A box contains 12 marbles of three different colours green, yellow and blue- four in each. If you were to close
your eyes and pick them at random, how many marbles must you take out to be sure at there are at least two of
one colour among the marbles picked out?
(a) 2 (b) 3 (c) 4 (d) 5

16. Michael lives 10 km way from where I live. Ahmed lives 5km away and Susan lives 7 km away from where
I live. Arun is farther away than Ahmed but closer than Susan from where I live. From the information
provided here, what is one possible distance (in km) at which I live from Arun’s place?
(EC/ME – 2016) (1M)
(a) 3.00 (b) 4.99 (c) 6.02 (d) 7.01

17. Two trains start from two opposite directions towards each other. The stations from which they start are 50 miles
apart. Both the trains start at the same time on a single track. A falcon which is sitting on one train, starts at the
same time towards other train, as soon as it reaches the second one, it flies back to the first train and so on and
so forth. It continues to do so, flying backwards and forwards from one train to the other until trains meet. Both
the trains travel at a speed of 25 miles per hour, and the bird flies at 100 miles per hour. How many miles will the
falcon have flown before the trains meet?
(a) 25 miles (b) 50 miles (c) 75 miles (d) 100 miles

18. A businessman advertised two job openings for peons in his firm. Two men applied and the businessman decided
to engage both of them. He offered them a salary of ₹.2, 000 per year; ₹.1000 to be paid every half year, with a
promise that their salary would be raised if their work proved satisfactory. They could have a raise of ₹.300 per
year, (or) if they preferred, ₹.100 each half year. The two men thought for a few moments and then one of them
expressed his wish to take the raise at ₹.300 per year, while the other man said he would accept the half yearly
increase of ₹.100. Between the two men, who was gainer?
(a) first person (b) second person
(c) Both are equal (d) None of these
4 Numerical Ability

19. Little Mammu was playing marbles with her friend Nawal I heard her say to him, if you give me one of your
marbles I’ll have as many as you. Nawal replied, if you give me one of your marbles, and I’ll have twice as many
as you. I wondered how many marbles each had? What do you think?
(a) 4 & 8 (b) 3 & 5 (c) 5 &7 (d) None of these

20. A father, I know, is 4 times his son’s age. And in 30 years son’s age will be half of his father’s age. How old are
the father and son now?
(a) 60, 30 (b) 90, 45 (c) 60, 45 (d) 60, 15

21. A square pyramid has a base perimeter x, and the slant height is half of the perimeter. What is the lateral surface
area of the pyramid?
(CE – 2016) (2M)
(a) x2 (b) 0.75 x2 (c) 0.50 x2 (d) 0.25 x2

22. A man owns a horse, a cow and a sheep. He also owns a pasture. If the horse and cow can eat the contents of the
pasture in 40 days, while the horse and sheep can do it in 60 days and the cow and the sheep in 90 days, how long
all of them together will take to eat all the contents?

19
(a) 360 (b) 360
19
(c) 720 19
(d) 1440 720

23. Ananth takes 6 hours and Bharath takes 4 hours to read a book. Both started reading copies of the book at the
same time. After how many hours is the number of pages to be read by Ananth, twice that to be read by Bharath?
Assume Ananth and Bharath read all the pages with constant pace.
(CE – 2016) (2M)
(a) 1 (b) 2 (c) 3 (d) 4

24. A cubic object 3″× 3″× 3″ is painted blue on all the outside surfaces, including the top and bottom. If the cube is
cut into 27 cubes of 1″ × 1″ × 1″ , how many 1″ cubes do have any painted surfaces.
(a) 6 (b) 16 (c) 26 (d) 27

25. A and B are friends. They decide to meet between 1 PM and 2 PM on a given day. There is a condition that who-
ever arrives first will not wait for the other for more than 15 minutes. The probability that they will meet on that
day is (EC/EE/INST-2012)(2M)
(a) 1/4 (b) 1/16 (c) 7/16 (d) 9/16

26. The variable cost (V) of manufacturing a product varies according to the equation V = 4q , where q is the quantity
produced. The fixed cost (F) of production of same product reduces with q according to the equation F = 100/q.
How many units should be produced to minimize the total cost (V+F) =?
(ME/CE/CSE/PI-2011)(2M)
(a) 5 (b) 4 (c) 7 (d) 6
5 Numerical Estimation

27. The cost function for a product in a firm is given by 5q2, where q is the amount of production. The firm can sell
the product at a market price of ₹. 50 per unit. The number of units to be produced by the firm such that the profit
is maximized is (ME/CE/CSE/PI-2011)(1M)
(a) 5 (b) 10 (c) 15 (d) 25

28. The cost of 7 pens, 8 pencils and 3 sharpeners is Rs.20. The cost of 3 pencils, 4 sharpeners and 5 erasers is Rs
21. The cost of 4 pens, 4 sharpeners, and 6 erasers is Rs 25. What is the cost of 1 pen, 1 pencil, 1 sharpener and 1
eraser?
(a) Rs. 11 (b) Rs. 13 (c) Rs. 6 (d) Cannot be determined

29. There are some oranges and mangoes in a basket and there are a certain number of plates in a cupboard. If each
orange is put in one plate, one orange is left without a plate to put it in. If two mangoes are put in each plate, then
one plate is left without any mango to put in it. If the difference between the number of oranges and the number
of mangoes is 3. What is the number of plates in the cupboard?
(a) 6 (b) 8 (c) 9 (d) Cannot be determined

30. There are a certain number of chocolates in a box. Ajit takes 1 chocolate less than half the number of chocolates
in the box. Burman takes 2 chocolates less than one-third of the remaining and then, Chanti takes 3 chocolates
less than one-fourth of the remaining. If there are still 36 chocolates left in the box, what was the initial number
of chocolates in the box?
(a) 63 (b) 124 (c) 186 (d) None of these

KEY For Logical Puzzles


01. (c) 02. (d) 03. (d) 04. (c) 05. (c) 06. (b) 07. (7)
08. (b) 09. (a) 10. (a) 11. (b) 12. (b) 13. (d) 14. (b)
15. (c) 16. (c) 17. (d) 18. (b) 19. (c) 20. (d) 21. (d)
22. (c) 23. (c) 24. (c) 25. (c) 26. (a) 27. (a) 28. (c)
29.(a) 30.(b)
Chapter 2 Numerical Reasoning
2.1. Number, Time Sequence Test , Ranking & Comparison

01. In the following series, how many such odd numbers are there which are divisible by 3 or 5, then
followed by odd numbers and then also followed by even numbers ?
12, 19, 21, 3, 25, 18, 35, 20, 22, 21, 45, 46, 47, 48, 9, 50, 52, 54, 55, 56
(a) Nil (b) One
(c) Two (d) Three

02. The letters L, M, N, O, P, Q, R, S and T in their order are substituted by nine integers 1 to 9 but not in that order.
4 is assigned to P. The difference between P and T is 5. The difference between N and T is 3. What is the integer
assigned to N? (CSE – 2016)
(a) 4 (b) 5 (c) 6 (d) 7

03. In the following sequence of instructions, 1 stands for Run, 2 stands for stop, 3 stands for Go, 4 stands for Sit and
5 stands for Wait. If the sequence were continued, which instruction will come next ?
44545345314531245453453
(a) Wait (b) Sit (c) Go (d) Run

04. Manisha ranked sixteenth from the top and twenty ninth from the bottom among those who passed an exam-
ination. Six boys did not participate in the competition and five failed in it. How many persons were there in the
class?
(a) 40 (b) 44 (c) 50 (d) 55

05. In a row of girls, Rita and Monika occupy the ninth place from the right end and tenth place from the left end,
respectively. If they interchange their places, Rita and Monika occupy seventeenth place from the right and eigh-
teenth place from the left, respectively. How many girls are there in the row ?
(a) 25 (b) 26 (c) 27 (d) Data inadequate

06. In a row of boys, Kapil is eighth from the right and Nikunj is twelfth from the left. When Kapil and Nikunj inter-
change positions, Nikunj becomes twenty first from the left. Which of the following will be Kapil’s position from
the right?
(a) 8th (b) 17th (c) 21st (d) Cannot be determined

07. In a queue, Amrita is 10th from the front while Mukul is 25th from behind and Mamta is just in the middle of the
two. If there be 50 persons in the queue, what position does Mamta occupy from the front ?
(a) 20th (b) 19th (c) 18th (d) 17th
7 Numerical Reasoning

08. Three persons A,B and C are standing in a queue. There are five persons between A and B and eight persons be-
tween B and C. If there be three persons ahead of C and 21 persons behind A, what could be the minimum number
of persons in the queue ?
(a) 41 (b) 40 (c) 28 (d) 27

09. Examine the following statements


I. Rama scored more than Rani
II. Rani scored less than Ratna
III. Ratna scored more than Padma
Iv. Padma scored more then Rama but less than Ratna
Who scored the highest ?
(a)Rama (b) Padma (c) Rani (d) Ratna

10. Five numbers 10, 7, 5, 4 and 2 are to be arranged in a sequence from left to right following the directions given
below:
1. No two odd or even numbers are next to each other.
2. The second number from the left is exactly half of the left-most number.
3. The middle number is exactly twice the right-most number.

Which is the second number from the right? (GATE-19)


(a) 2 (b) 4 (c) 7 (d) 10

11. There are six persons A,B,C,D,E and F
I. A has 3 items more than C.
II. D has 4 items less than B.
III. E has 6 items more than F.
IV. C has 2 items more than E.
V. F has 3 items more than D.
Which of the following figures cannot be equal to the total number of items possessed by all the 6 persons?
(a) 41 (b) 47 (c) 53 (d) 58

12. Ajay left home for the bus stop 15 minutes earlier than usual. It takes 10 minutes to reach the stop. He reached the
stop at 8.40 a.m. What time does he usually leave home for the bus stop?
(a) 8.30 a.m. (b) 8.45 p.m. (c) 8.55 a.m. (d) None of these

13. The train for Lucknow leaves every two and a half hours from New Delhi Railway station. An announcement was
made at the station that the train for lucknow had left 40 minutes ago and the next train will leave at 18.00 hrs. At
what time was the announcement made?
(a) 15.30 hrs (b) 17.10 hrs
(c) 16.00 hrs (d) None of these
8 Numerical Ability

14. An application was received by inward clerk in the afternoon of a week day. Next day he forwarded it to the table
of the senior clerk, who was on leave that day. The senior clerk next day evening put up the application to the desk
officer. Desk officer studied the application and disposed off the matter on the same day i.e., Friday. Which day
was the application received by the inward clerk?
(a) Monday (b) Tuesday
(c) Wednesday (d) None of these

15. There are twenty people working in an office. The first group of five works between 8.00 A.M. and 2.00 P.M. The
second group of ten works between 10.00 A.M. and 4.00 P.M. And the third group of five works between 12 noon
and 6.00 P.M. There are three computers in the office which all the employees frequently use. During which of the
following hours the computers are likely to be used most?
(a) 10.00 A.M. – 12 noon (b) 12 noon – 2.00 P.M.
(c) 1.00 P.M. – 3.00 P.M (d) 2.00 P.M. – 4.00 P.M

16. A monkey climbs 30 feet at the beginning of each hour and rests for a while when he slips back 20 feet before he
again starts climbing in the beginning of the next hour. If he begins his ascent at 8.00 a.m., at what time will he
first touch a flag at 120 feet from the ground?
(a) 4 p.m. (b) 5 p.m. (c) 6 p.m (d) None of these

17. Mohini went to the movies nine days ago. She goes to the movies only on Thursday. What day of the week is
today?
(a) Thursday (b) Saturday (c) Sunday (d) Tuesday

18. If 3rd December, 1990 is Sunday, what day is 3rd January, 1991?
(a) Tuesday (b)Wednesday (c) Thursday (d) Friday

19. If 18th February, 1997 falls on Tuesday then what will be the day on 18th February, 1999?
(a) Monday (b) Tuesday (c) Thursday (d) Friday

20. How many days will there be from 26th January, 1996 to 15th May, 1996 (both days included)?
(a) 110 (b) 111 (c) 112 (d) 113

KEY for Number, Time Sequence Test, Ranking & Comparison

01. (c) 02. (c) 03. (d) 04. (d) 05. (b) 06. (b) 07. (c) 08. (c) 09. (d) 10. (c)

11. (d) 12. (d) 13. (d) 14. (c) 15. (b) 16. (c) 17. (b) 18. (b) 19. (c) 20. (b)
9 Numerical Reasoning

2.2 Directions & Distances

01. Ganguly went 15 km to the West from my house then turns left and walked 20 km. He then turned East and walked
25 km and finally turned left and covered 20 km. How far was he from my house?
(a) 5 Kms (b) 10 Kms (c) 40 Kms (d) 80 Kms

02. From his house Lokesh went 15 Km to the North. Then he turned West and covered 10 Kms. Then he turned South
and covered 5 Km. Finally turning to east, he covered 10 Kms. In which is he from his house?
(a) West (b) East (c) North (d) None

03. Ravi walks 10 mts towards the South turning to the left he walks 20 mts and then moves to his right. After
moving distance of 20 mts he turns to the right and walks 20 mts. Finally he turns to right and moves an distance of
10 mts. How far and in which direction is he from the starting point?
(a) 10 mts North (b) 20 mts South (c) 20 mts North (d) 10 mts South

04. Going 50 m to the South of her house. Radhika turns left and goes another 20 m. Turning to the North she goes
30 m and then starts walking to her house. In which direction is she walking now?
(a) North – West (b) North (c) South – East (d) East

05. Facing the East Mr. U.V. Rao turned left and walked 10 mts then turned to his left again and walked 10 mts. He
then turned 450 towards his right and went straight to cover 25 mts. In which direction from his starting point is
he?
(a) South – West (b) South – East (c) North – West (d) North – East

06. Preeti wants to go to the market. She starts from her home which is in the North and comes to the crossing, the
road to her left ends in a park and straight ahead is the office complex. In which direction is the market?
(a) East (b) West (c) North (d) South

07. I start from my home and go 2 km straight. Then if turn to words my right and go 1 Km. I turn again towards my
right and go 1 Km again. If I am North – West from my house, then in which direction did I go in beginning?
(a) North (b) West (c) South (d) East

08. A, B, C and D are playing a game of caroms. A, C and B, D are partners. D is to the right of C who is facing West.
There B is facing in
(a) North (b) South (c) East (d) West

09. Lokesh’s school bus is facing North when it reaches his school. After starting from Lokesh house, it turns right
twice and then left before reaching the school. What direction was the bus facing when if left the bus stop in front
of Lokesh’s house.
(a) North (b) South (c) East (d) West
10 Numerical Ability

10. A walk towards North 4 Kms and turns right and walks 6 Kms. Then he turns towards South and walks 2 Kms.
Again he takes a turn towards West walks 3 Kms and stops a while. Then we further walk 3 Kms. What is the
distance of A from his starting point?
(a) 16 Kms (b) 12 Kms (c) 2 Kms (d) 4 Kms

11. Rita drives to the North of her place of stay at A and after traveling 25 km, finds that she has driven in the wrong
direction she then turns to her right and travels 2 km and then she again turns to the right and drives straight
another 25 km. How much distance has she now to cover to go back to the starting point?
(a) 25 km (b) 2 km (c) 4 km (d) 40 km

12. Rana travels 10 km to the North, turns left and travels 4 km, and then again turns right and covers another 5 km,
and then turns right, and travels another 4 km. How far is he from the right starting point.
(a) 15 km (b) 4 km (c) 5 km (d) 10 km

13. A taxi driver commenced his journey from a point, and drove 10 km towards North, and turned to his left and
drove another 5 km. After waiting to meet a friend here, he turned to his right and continued to drive another 10
km. He has covered a distance of 5 km to East then, in which direction would he be now
(a) North (b) East (c) South (d) West

14. ‘Barin’ village is 20 kilometers to the north of village ‘Khanof’ village ‘Banoha’ is 18 kilometers to the east of
village ‘Khanof’ village ‘Palasi’ is 12 kilometers to the west of ‘Barin’, If Amitabh starts from village Banoha and
goes to village palasi in which direction is he from his starting point?
(a) North – East (b) North – West
(c) South – East (d) North

15. I am facing east. I turn 1000 in the clock wise direction and then 450 in the anticlock wise direction. Which direc-
tion am I facing now?
(a) North - West (b) South – East (c) North - East (d) South – West

16. A watch reads 4:30. If the minute hand points East, in what direction will the hour hand point?
(a) North - East (b) South - West (c) North (d) South

17. A clock is so placed that at 12 noon its minute hand points towards North-East. In which direction does its hour
hand point at 1.30 p.m.?
(a) North (b) South (c) East (d) West

18. Ankit, Tarun, Rohan and Sohan are friends. They play cards. Partner faces in opposite directions. Ankit and Tarun
become parterns. Sohan faces North. If Ankit faces towards West, then who faces towards South?
(a) Tarun (b) Rohan (c) Sohan (d) Data inadequate
11 Numerical Reasoning

19. Meena wants to go to the market. She starts from her home which is towards North and comes to the crossing.
The road to her left ends in a park and straight ahead is the office complex. In which direction is the market from
the crossing if the four places lie in different directions?
(a) East (b) West (c) North (d) South

20. A is 40 m South-West of B, C is 40 m South-East of B. Then, C is in which direction with respect to A?


(a) East (b) West (c) North-East (d) South

KEY for Directions & Distances

01. (b) 02. (c) 03. (b) 04. (a) 05. (c) 06. (b) 07. (b) 08. (a) 09. (d) 10. (c)
11. (b) 12. (a) 13. (a) 14. (b) 15. (b) 16. (a) 17. (c) 18. (b) 19.(b) 20.(a)

2.3 Arithmetical Reasoning

01. A bird shooter was asked how many birds he had in the bag. He replied that there were all sparrows but six, all
pigeons but six, and all docks but six. How many birds had he in all?
(a) 9 (b) 18 (c) 27 (d) 36

02. In a cricket match, five batsmen A, B, C, D and E scored an average of 36 runs. D scored 5 more than E; E scored
8 fewer than A; B scored as many as D and E combined; and B and C scored 107 between them. How many runs
did E score?
(a) 62 (b) 45 (c) 28 (d) 20

03. A, B, C, D and E play a game of cards. A says to B, “If you give me three cards, you will have as many as E has
and if I give you three cards, you will have as many as D has.” A and B together have 10 cards more than what D
and E together have. If B has two cards more than what C has and the total number of cards be 133, how many
cards does B have?
(a) 22 (b) 23 (c) 25 (d) 35

04. A, B, C, D and E play a game of cards. A says to B., “If you give me 3 cards, you will have as many as I have at
this moment while if D takes 5 cards from you, he will have as many as E has”. A and C together have twice as
many cards as E has. B and D together also have the same number of cards as A and C taken together. If together
they have 150 cards, how many cards has C got?
(a) 28 (b) 29 (c) 31 (d) 35
12 Numerical Ability

05. A certain number of horses and an equal number of men are going somewhere. Half of the owners are on their
horses’ back while the remaining ones are walking along leading their horses. If the number of legs walking on
the ground is 70, how many horses are there?
(a) 10 (b) 12 (c) 14 (d) 16

06. In a certain office, 13 of the workers are women, 12 of the women are married and 13 of the married

women have children. If 34 of the men are married and 3


2
of the married men have children, what

part of workers are without children?



5
(a) 18 (b) 94 (c) 11 18
(d) 17
36

07. In a family, a couple has a son and a daughter. The age of the father is three times that of his daughter and the age
of the son is half of his mother. The wife is 9 years younger to her husband and the brother is seven years older
than his sister. What is the age of the mother?
(a) 40 years (b) 45 years
(c) 50 years (d) 60 years

08. Ravi’s brother is 3 years senior to him. His father was 28 years of age when his sister was born while his mother
was 26 years of age when he was born. If his sister was 4 years of age when his brother was born, what was the
age of Ravi’s father and mother respectively when his brother was born?
(a) 32 years, 23 years (b) 32 years, 29 years
(c) 35 years, 29 years (d) 35 years, 33 years

09. When Rahul was born, his father was 32 years older than his brother and his mother was 25 years older than his
sister. If Rahul’s brother is 6 years older than him and his mother is 3 years younger than his father, how old was
Rahul’s sister when he was born?
(a) 7 years (b) 10 years (c) 14 years (d) 19 years

10. In a sports academy of 300 people, 105 play only cricket, 70 play only hockey, 50 play only football, 25 play both
cricket and hockey, 15 play both hockey and football and 30 play both cricket and football. The rest of them play
all three all three sports. What is the percentage of people who play at least two sports?
(GATE - 2019)
(a) 28.00 (b) 23.30 (c) 50.00 (d) 25.00

11. Forty students watched films A, B and C over a week. Each student watched either only one film or all three.
Thirteen students watched film A, sixteen students watched film B and nineteen students watched film C. How
many students watched all three films?
(GATE - 2018)
(a) 0 (b) 2 (c) 4 (d) 8
13 Numerical Reasoning

12. Consider the Venn diagram given below :


Paper I Paper II

3 12 4

2
8 5

Paper III

The number in the Venn diagram indicates the number of persons reading the newspapers. The diagram is drawn
after surveying 50 persons. In a population of 10,000, how many can be expected to read at least two newspapers?
(a) 5000 (b) 5400 (c) 6000 (d) 6250

13. Out of a total of 120 musicians in a club, 5% can play all the three instruments – guitar, violin and flute. It so hap-
pens that the number of musicians who can play any two and only two of the above instruments is 30. The number
of musicians who can play the guitar alone is 40. What is the total number of those who can play violin alone or
flute alone?
(a) 30 (b) 38 (c) 44 (d) 45

14. 500 students are taking one or more courses out of chemistry, physics and Mathematics. Registration records
indicate course enrolment as follows: chemistry (329), physics (186), Mathematics (295), chemistry and physics
(83), chemistry and Mathematics (217), and physics and Mathematics (63), How many students are taking all 3
subjects ? (EC-2017)
(a) 37 (b) 43 (c) 47 (d) 53

15. A number of friends decided to go on a picnic and planned to spend Rs. 96 on eatables. Four of them, however,
did not turn up. As a consequence, the remaining ones had to contribute Rs. 4 each extra. The number of those
who attended the picnic was
(a) 8 (b) 12 (c) 16 (d) 24

KEY for Arithmetical Reasoning

01. (a) 02. (d) 03. (c) 04. (a) 05. (c) 06. (c) 07. (d) 08. (a) 09. (b) 10. (d)
11. (c) 12. (b) 13. (c) 14. (d) 15. (a)
14 Numerical Ability

2.4 Blood Relations

01. Anil introduces Rohit as the son of the only brother of his father’s wife. How is Rohit related to Anil ?
(a) Cousin (b) Son (c) Uncle (d) Son-in-law

02. Pointing towards a person in a photograph, Anjali said, “He is the only son of father of my sister’s brother”. How
is that person related to Anjali?
(a) Father (b) Maternal uncle
(c) Cousin (d) None of these

03. Pointing out to a photograph a man tells his friend. “She is the daughter of the only son of my father’s wife”. How
is the girl in the photograph related to the man?
(a) Daughter (b) Cousin (c) Mother (d) Sister

04. Pointing to a man on the stage, Rita said, “He is the brother of the daughter of the wife of my husband”. How is
the man on the stage related to Rita ?
(a) Son (b) Husband (c) Cousin (d) Nephew

05. If Kamal says, Ravi’s mother is the only daughter of my mother”. How is kamal related to Ravi ?
(a) Grand father (b) Father
(c) Cannot be determined (d) None of these

06. Pointing to a man in a photograph, Asha, said, “His mother’s only daughter is my mother”. How is Asha related
to that man ?
(a) Nephew (b) Sister (c) Wife (d) Niece

07. Pointing to a man in a photograph, a woman said, “His brother’s father is the only son of my grand father”. How
is the woman related to the man in the photograph?
(a) Mother (b) Aunt (c) Sister (d) Daughter

08. Study the following information carefully and answer the question given below it.
There are six persons, A, B, C, D, E and F. C is the sister of F, B, is the brother of E’s Husband. D is the father of
A and grandfather of F. There are two fathers, three brothers and a mother in the group.

(i) Who is the mother?


(a) A (b) B (c) C (d) E

(ii) Who is E’s husband ?


(a) B (b) C (c) A (d) F
15 Numerical Reasoning

(iii) How many male members are there in the group ?


(a) One (b) Two (c) Three (d) Four

(iv) How is F related to E ?


(a) Uncle (b) Husband (c) Son (d) Daughter

(v) Which of the following is a group of brother


(a) ABF (b) ABD (c) BFC (d) BDF

09. M has a son Q and a daughter R. He has no other children. E is the mother of P and daughter-in law of M. How is
P related to M?
(EC/IN – 2016)
(a) P is the son-in-law of M (b) P is the grandchild of M
(c) P is the daughter in law of M (d) P is the grandfather of M

10. Each of P,Q, R,S, W,X,Y and Z has been married at most once. X and Y married and have two children P and Q
.Z is the grandfather of the daughter S of P. Further, Z and W are married and are parents of R. Which one of the
following must necessarily be FALSE ?
(EC-2017)
(a) X is the mother in law of R (b) P and R not married to each other
(c) P is a son of X and Y (d) Q cannot be married to R

11. P, Q, R, S and T are related and belong to the same family. P is the brother of S. Q is the wife of P. R and T are the
children of the siblings P and S respectively. Which one of the following statements is necessarily FALSE?
(GATE-19)
(a) S is the sister-in-law of Q (b) S is the brother of P
(c) S is the aunt of T (d) S is the aunt of R

12. M and N had four children P, Q, R and S. Of them, only P and R were married. They had children X and Y
respectively. If Y is a legitimate child of W, which one of the following statements is necessarily FALSE?
(a) W is the wife of P (b) W is the wife of R (GATE-19)
(c) M is the grandmother of Y (d) R is the father of Y

13. Read the following information carefully and answer the questions that follow : A + B means. A is the son of B,
A – B means A is the wife of B, A x B means A is brother of B, A÷ B means A is the mother of B and A = B means
A is the sister of B.

(i) What does P + R – Q mean ?


(a) Q is the father of P (b) Q is the son of P
(c) Q is the uncle of P (d) Q is the brother of P

16 Numerical Ability

(ii) What does P x R ÷ Q mean ?


(a) P is the brother of Q (b) P is the father of Q
(c) P is the uncle of Q (d) P is the nephew of Q

(iii) What does P = R + Q mean?


(a) P is the aunt of Q (b) P is the daughter of Q
(c) P is the niece of Q (d) P is the sister of Q

(iv) What does P = R ÷ Q mean ?


(a) P is the aunt of Q (b) P is the sister of Q
(c) Q is the niece of P (d) Q is the daughter of P

14. Read the following information carefully and answer the questions given below.
A + B means A is the daughter of B, A x B means A is the son of B and A – B means A is the wife of B.

(i) If P x Q – S, which of the following is true?


(a) S is wife of Q (b) S is father of P
(c) P is daughter of Q (d) Q is father of P

(ii) If T – S x B – M, Which of the following is not true ?


(a) B is mother of S (b) M is husband of B
(c) T is wife of S (d) S is daughter of B

(iii) If Z + T – S x U + P, What is U to Z.
(a) Mother (b) Grand mother
(c) Father (d) Cannot be determined

15. P + Q means P is the brother of Q, P – Q means P is the mother of Q and P x Q means P is the sister of Q. Which
of the following means M is the maternal uncle of R.
(a) M + K + R (b) M – R + K (c) M + K – R (d) M + K x R

KEY for Blood Relations

01. (a) 02. (d) 03. (a) 04. (a) 05. (d) 06. (d) 07. (c) 08.i.(d) ii. (c) iii.(d)

iv. (c) v. (a) 09. (b) 10. (b) 11. (c) 12. (a) 13.i. (a) ii. (c) iii. (b) iv. (a)

14.i. (b) ii. (d) iii. (b) 15. (c)


17 Numerical Reasoning

2.5 Cubes & Dice

01. If x figure is folded then which of the following cube is formed?


2
5 1
4
6 3

(X)
1 2 3
3
1 5 4 4
6 3 6 1

(a) (b) (c) (d)


02. When the following figure is folded to form a cube, how many dots would lie opposite the face bearing five
dots?

(a) 1 (b) 2 (c) 3 (d) 6

03. How many dots lie opposite the face having three dots, when the given figure is folded to form a cube?

(a) 2 (b) 4 (c) 5 (d) 6

04. The six faces of a cube have been marked with numbers 1, 2, 3, 4, 5 and 6 respectively. This cube is rolled down
three times. The three positions are given. Choose the figure that will be formed when the cube is unfolded.

6 1 5
4 3 6
2 2 3
18 Numerical Ability

2 5 6 1
3 4 3 6 2 4 2 3
1 1 1 6
6 5 2 4 3 5 5 4

(a) (b) (c) (d)


05. Which number is on the face opposite 6 ?

6 6 5 1
3 2 2 4 6 4 4 2

(i) (ii) (iii) (iv)

(a) 1 (b) 2 (c) 3 (d) 4

06. Which number is on the face opposite 4?

2 1 4 2
5 1 5 6 6 5 4 3

(i) (ii) (iii) (iv)

(a) 1 (b) 2 (c) 3 (d) 5

07. Which number is opposite 3?

5 2 3 4
4 6 1 4 6 5 1 5


(i) (ii) (iii) (iv)

(a) 1 (b) 2 (c) 4 (d) 6

08. What numbers occur at the bottom face in the three positions of the same die?

4 4 4

2 3 1 2 3 6

(i) (ii) (iii)


(a) 6, 6, 2 (b) 5, 6, 1 (c) 5, 5, 5 (d) 6, 5, 2
19 Numerical Reasoning

09.
5 4 2

4 1 6 3 6 5

(X) (Y) (Z)



(i) Which number lies at the bottom face of the die X ?
(a) 1 (b) 2 (c) 3 (d) 4
(ii) Which number lies at the bottom face of the die Y ?
(a) 6 (b) 5 (c) 2 (d) 1
(iii) Which number lies opposite 6 ?
(a) 1 (b) 2 (c) 4 (d) 5
(iv) Which numbers are hidden behind the numbers 6 and 5 in the die Z?
(a) 1 & 4 (b) 1 & 3 (c) 4 & 3 (d) 1 & 2
(v) Which of the hidden numbers adjacent to 5 in die X are common to the hidden numbers adjacent to 5 in die
Z?
(a) 1 & 4 (b) 2 (c) 6 (d) None

10. What will be the number at the bottom if 5 is at the top; the two positions of the dice being as given below:

1 6

4 2 2 3

(i) (ii)

(a) 1 (b) 2 (c) 3 (d) 6

KEY for Cubes & Dice

01. (d) 02. (c) 03. (d) 04. (c) 05. (a) 06. (a) 07. (c) 08. (c) 09.i. (c) ii. (c)

iii. (a) iv. (b) v. (d) 10. (b)


20 Numerical Ability

2.6 Coding and Decoding Tests


01. If in a certain language MYSTIFY is coded as NZTUJGZ, how is NEMISES coded in that code?
(a) OFNJTFT (b) MDLHRDR
(c) ODNHTDR (d) PGOKUGU

02. If in certain language, NATURE is coded as OCUWSG, how is FAMINE coded in that code?
(a) GCNKOF (b)GCNJOF
(c) GCNKOG (d) HCNKOG

03. In a certain code, COMPUTER is written as FPSOVUHS. How is MEDICINE written in the same code?
(a) PFFDJOF (b) NFFDJOF
(c) LFFDJOF (d) PFJHDJQF

04. In a certain code, MONKEY is written as XDJMNL. How is TIGER written in that code?
(a) QDFHS (b) SDFHS (c) SHFDQ (d) UJHFS

05. In a certain code, AMCF is written as EQGJ and NKUF is written as ROYJ. How will DHLP be written in the
code? (GATE - 2018)
(a) RSTN (b) TLPH (c) HLPT (d) XSVR

06. If IMHO = JNIP; IDK =JEL: and SO =TP, then IDC = ____. (GATE - 2019)
(a) JDC (b) JCD
(c) JDE (d) JED

07. If 7,2 = 59; 5,3 = 28; 9,1 = 810; 2,1= 13; 5,4 = ?
(a) 19 (b) 9 (c) 20 (d) 239

08. If 137+276 = 435, how much is 731+672 =? (GATE – 2010)


(a) 534 (b) 1403 (c) 1623 (d) 1513

09. If 234+567 = 1023, how much is 432+765=?


(a) 3201 (b) 1417 (c) 1714 (d) 1516

10. If 4444+5555 = 12221, how much is 5555+6666=?
(a) 1331 (b) 14441 (c) 14443 (d) 44431

11. If 4567+3456 = 10245, how much is 7654+6543 = ?


(a) 16417 (b) 17416 (c) 411617 (d) 16174
21 Numerical Reasoning

12. If REASON is coded as ‘5’ and BELIEVED as 7, What is the code number for GOVERNMENT?
(a) 6 (b) 8 (c) 9 (d) 10

13. LUNCH = 6, DINNER = 8, SUPPER = 8, Then BREAKFAST =?


(a) 8 (b) 12 (c) 14 (d) 16

14. If D = 4 and COVER = 63, Then BASIS = ?


(a) 49 (b) 50 (c) 54 (d) 55

15. If Z = 52 and ACT = 48, Then BAT will be equal to


(a) 89 (b) 41 (c) 44 (d) 46

16. If GO = 32, SHE = 49, Then SOME will be equal to


(a) 56 (b) 58 (c) 62 (d) 64

17. If AT = 20, BAT = 40, Then CAT will be equal to


(a) 30 (b) 50 (c) 60 (d) 70

18. If E = 10; J = 20; O = 30; and T = 40, what will be P + E + S + T? (GATE - 2019)
(a) 120 (b) 164 (c) 82 (d) 51

19. If CAP = 6, and NO = 6 then QBA = ?


(a) 9 (b) 8 (c) 6 (d) 4

20. If BANANA = 27, and MOTORE = 16, then LOFERS=?
(a) 5 (b) –5 (c) 3 (d) –3

21. If A = 26 and BG = 18 then PAC = ?
(a) 7 (b) 20 (c) 25 (d) 6

22. If AROMA = 24, GRAND = 22, KWALITY = ?
(a) 40 (b) 62 (c) 55.5 (d) 50.5

23. If PLANT is coded as 12.60, BARS is coded as 10, what will be the code for LEAVES?
(a) 11 (b) 10 (c) 10.5 (d) 10.66

24. If MACHINE is coded as 19-7-9-14-15-20-11, how will you code DANGER?


(a) 10 – 7 – 20 – 13 – 11 – 24 (b) 11 – 7 – 20 – 16 – 11 – 24
(c) 13 – 7 – 20 – 9 – 11 – 25 (d) 13 – 7 – 20 – 10 – 11 – 25
22 Numerical Ability

25. If MANGO is coded as 31-1-41-7-51, how will JUHI be coded?


(a) 10-21-8-9 (b) 01-12-8-9
(c) 10-12-8-9 (d) 01-21-8-9

26. If white is called blue, blue is called red, red is called yellow, yellow is called green, green is called black, black
is called violet and violet is called orange, what would be the colour of human blood?
(a) Red (b) Green
(c) Yellow (d) Violet

27. If room is called bed, bed is called window, window is called flower and flower is called cooler, on what would a
man sleep?
(a) Window (b) Bed
(c) Flower (d) Cooler

28. If Sand is called air, air is called Rock, Rock is called well, well is called Drainage and Drainage is called sky,
Then from where will a woman draw water?
(a) Well (b) Drainage
(c) Sky (d) Air

29. In a certain code ‘bi nie pie’ means ‘some good Jokes’ ‘nie bat lik’ means ‘some real’ stories and ‘pie lik to’ means
‘many good stories’ which word in that code means Jokes?
(a) bi (b) nie
(c) pie (d) Can not be determined

30. In a certain language ‘pic vic nic ‘ means ‘winter is cold; ‘to nic re’ means ‘summer is hot’; ‘re it pa’ means
‘nights are hot’;
Which of the following is the code for ‘summer’?
(a) to (b) nic
(c) pic (d) vic

KEY for Coding and Decoding Tests

01. (a) 02. (c) 03. (d) 04. (a) 05. (c) 06. (d) 07. (a) 08. (c) 09. (b) 10. (c)
11. (a) 12. (c) 13. (b) 14. (b) 15. (d) 16. (a) 17. (c) 18. (a) 19. (c) 20. (d)
21. (a) 22. (d) 23. (d) 24. (a) 25. (b) 26. (c) 27. (a) 28. (b) 29. (a) 30. (a)
23 Numerical Reasoning

2.7 Inserting the Missing Character

01. Find the missing character from among the given alternatives

3 15

2 6
25 441
289 ?
10 13
7 12

(a) 625 (b) 25 (c) 125 (d) 156

02.
2 6
3

3 2 4 54 3
33 2 ? 5

4 5 4

(a) 78 (b) 82 (c) 94 (d) 86

03.

25 17 38 18 89 16

6 8 ?

(a) 13 (b) 15 (c) 17 (d) 19

04.
6
1 4
8 3
9 5 5 3

651 246 ?


3 4 7 9 2 1
6 8 4

(a) 262 (b) 622 (c) 631 (d) 824


24 Numerical Ability

05.

93 27 79 38 67 16

63 37 42

3 4 ?

(a) 5 (b) 6 (c) 8 (d) 9

06.
406 ?

81 5

3
16

(a) 1 (b) 731 (c) 1625 (d)2031

07.

13
1
? 4 2 5
3
8

(a) 10 (b) 11 (c) 12 (d) 13

08.
15 17 6 2 23 28
40 28 ?
9 3 12 13 4
8

(a) 52 (b) 51 (c) 1 (d) 2


25 Numerical Reasoning

09.

101 43 48 34
?
838 38

35 15 56 184

(a) 127 (b) 142 (c) 158 (d) 198

10.

7 3 5
3 5 4
39 51 ?
6 3 5 4 5 4

(a) 47 (b) 45 (c) 37 (d) 35

11.
15 2 9 7 13 16

80 65 ?

5 4 6 11 8
6

(a) 48 (b) 72 (c) 35 (d) 120

12.
3 7 1

6 25 2 11 70 8 4 -12 5

4 6 ?

(a) 10 (b) 6 (c) 2 (d) 1


26 Numerical Ability

13.
6 18 15

3 2 5

4 3 ?

8 27 9

(a) 3 (b) 4 (c) 6 (d) 8

14.

4 5 6

2 3 7

1 8 3

21 98 ?

(a) 98 (b) 96 (c) 94 (d) 80

15.
4C 2B 3A

28A ? 45B

7C 5A 15B

(a) 10c (b) 12c (c) 13c (d) 7c

KEY for Inserting the Missing Character

01. (a) 02. (d) 03. (b) 04. (b) 05. (d) 06. (d) 07. (c) 08. (c) 09. (b) 10. (c)
11. (a) 12. (c) 13. (a) 14. (c) 15. (a)
27 Numerical Reasoning

2.8 Analytical Reasoning

Directions for Questions 01 to 05


Read the following information carefully and answer the questions given below.
Six persons A, B, C, D, E and F are sitting in two rows, three in each.
E is not at the end of any row
D is second to the left of F
C, the neighbour of E, is sitting diagonally opposite to D
B is the neighbour of F.

01. Which of the following are sitting diagonally opposite to each other ?
(a) F and C (b) D and A (c) A and C (d) A and F

02. Who is facing B ?


(a) A (b) C (c) D (d) E

03. Which of the following are in the same row ?


(a) A and E (b) E and D (c) C and B (d) A and B

04. Which of the following are in one of the two rows ?


(a) FBC (b) CEB (c) DBF (d) AEF

05. After interchanging seat with E, who will be the neighbours of D in the new position ?
(a) C and A (b) F and B (c) only B (d) only A

Directions for Questions 06 to 10:


A training college has to conduct a refresher course for teachers of seven different subject Mechanics, Psychology,
Philosophy, Sociology, Economics, Science and Engineering from 22nd July to 29th July.
(i) Course should start with psychology
(ii) 23rd July, being Sunday, should be holiday
(iii) Science subject should be on the previous day of the Engineering subject
(iv) Course should end with Mechanics subject
(v) Philosophy should be immediately after the holiday.
(vi) There should be a gap of one day between Economics and Engineering.
(vii) There should be a gap of two days between Economics and Sociology.

06. The refresher course will start with which one of the following subjects?
(a) Psychology (b) Mechanics
(c) Philosophy (d) Economics
28 Numerical Ability

07. Which subject will be on Tuesday?


(a) Mechanics (b) Engineering (c) Economics (d) None of these

08. Which subject precedes Mechanics?


(a) Economics (b) Engineering (c) Philosophy (d) None of these

09. How many days gap is there between science and philosophy?
(a) One (b) Two (c) Three (d) No gap

10. Which subject is followed by science?


(a) Engineering (b) Psychology (c) Philosophy (d) Economics

Directions for Questions 11 to 15:


Seven friends Kamala, Manish, Rohit, Amit, Gaurav, Prem and Priya are sitting in a circle Kamala, Manish, Rohit,
Amit, Prem and Priya are sitting at equal distances from each other. Rohit is sitting two places right of Prem who
is sitting right of Amit but Rohit and Prem are not sitting opposite to each other. Kamala forms an angle of 900
from Gaurav and an angle of 1200 from Manish. Manish is just opposite Priya and is sitting on the left of Gaurav.

11. Who is the only person sitting between Rohit and Manish ?
(a) Prem (b) Amit (c) Gaurav (d) Kamala

12. Gaurav is not sitting at equal distances from __


(a) Rohit and Prem (b) Amit and Kamala
(c) Manish and Prem (d) All the above

13. Gaurav is sitting ___________ of Priya


(a) to the left (b) to the right (c) two places right (d) None of these

14. The angle between Gaurav and Manish in the clockwise direction is ______
(a) 1500 (b) 1800 (c) 2100 (d) None of these

15. Which of the following statements is not correct ?


(a) Prem is between Manish Kamala (b) Manish is two places away from Priya
(c) Gaurav is sitting opposite Prem (d) All of the above

16. There are five buildings called V, W, X, Y and Z in a row (not necessarily in that order). V is to the West of W, Z
is to the East of X and the West of V, W is the West of Y. Which is the building in the middle?
(EE-2017)
(a) V (b) W (c) X (d) Y
29 Numerical Reasoning

17. Rahul, Murali, Srinivas and Arul are seated around a square table. Rahul is sitting to the left of Murali. Srinivas is
sitting to the right of Arul. Which of the following pairs are seated opposite each other?
(EE-2017)
(a) Rahul and Murali (b) Srinivas and Arul
(c) Srinivas and Murali (d) Srinivas and Rahul

18. Six people are seated around a circular table. There are at least two men and two women. There are at least three
right-handed persons. Every woman has a left-handed person to her immediate right. None of the women are
right-handed. The number of women at the table is (EE-2017)
(a) 2 (b) 3 (c) 4 (d) Cannot be determined

19. P,Q,R,S,T and U are seated around a circular table , R is seated two places to the right of Q, P is seated three places
to the left of R.S is seated opposite U. If P and U now switch seats, which of the following must necessarily be
true? (CE-2017)
(a) P is immediately to the right of R
(b) T is immediately to the left of P
(c) T is immediately to the left of P or P is immediately to the right of Q
(d) U is immediately to the right of R or P is immediately to the left of T

20. S, T, U, V, W, X, Y, and Z are seated around a circular table. T’s neighbours are Y and V. Z is seated third to the
left of T and second to the right of S. U’s neighbours are S and Y; and T and W are not seated opposite each other.
Who is third to the left of V? (EC-2017)
(a) X (b) W (c) U (d) T

21. Leela is older than her cousin Pavithra, pavithra’s brother Shiva is older than Leela. When Pavithra and shiva are
visiting Leela, all three like to play chess. Pavithra wins more often than Leela does.
(EC/ME – 2016)
Which one of the following statements must be TRUE based on the above?
(a) When Shiva plays chess with Leela and Pavithra. he often loses.
(b) Leela is the oldest of the three.
(c) Shiva is a better chess palyer than Pavithra
(d) Pavithra is the youngest of the three

22. Consider five people- Mitra, Ganga, Rekha, Lakshmi and Sana, Ganga is taller than both Rekha and Lakshmi.
Lakshmi is taller than Sana. Mita is taller than Ganga. Which of the following conclusions are true?
1. Lakshmi is taller than Rekha
2. Rekha is shorter than Mita
3. Rekha is taller than Sana
4. Sana is shorter than Ganga (GATE – 19)
(a) 1 only (b) 1 and 3 (c) 3 only (d) 2 and 4
30 Numerical Ability

23. Four people are standing in a line facing you. They are Rahul, Mathew, Seema and Lohit. One is an engineer, one
is a doctor, one a teacher and another a dancer. You are told that:
1. Mathew is not standing next to Seema
2. There are two people standing between Lohit and the engineer
3. Rahul is not a doctor
4. The teacher and the dancer are standing next to each other
5. Seema is turning to her right to speak to the doctor standing next to her
Who among them is an engineer? (GATE – 19)
(a) Mathew (b) Rahul (c) Seema (d) Lohit

24. Consider the following statements relating to the level of poker play of four players P, Q, R and S.
(CE – 2016) (2M)
I. P always beats Q
II. R always beats S
III. S loses to P only sometimes
IV. R always loses to Q
Which of the following can be logically inferred from the above statements?
(i) P is likely to beat all the three other players
(ii) S is the absolute worst player in the set
(a) (i) only (b) (ii) only
(c) (i) and (ii) (d) neither (i) nor (ii)

25. A flat is shared by four first year undergraduate students. They agreed to allow the oldest of them to enjoy some
extra space in the flat. Manu is two months older than Sravan, who is three months younger than Trideep.Pavan is
one month older than Sravan.Who should occupy the extra space in the falt?
(EC/IN – 2016) (2M)
(a) Manu (b) Sravan (c) Trideep (d) Pavan

KEY for Analytical Reasoning

01. (d) 02.(d) 03. (a) 04. (c) 05. (a) 06. (a) 07. (c) 08. (d) 09. (a) 10. (a)
11. (c) 12. (d) 13. (d) 14. (d) 15. (d) 16. (a) 17. (c) 18. (a) 19. (c) 20. (a)
21. (d) 22. (d) 23. (a) 24. (a) 25. (c)
31 Numerical Reasoning

2.9 Analytical Figures

01. How many straight lines are there in the following figure?


(a) 9 (b) 11 (c) 15 (d) 48

02. What is the number of straight lines and the number of triangles in the given figure?

(a) 10 straight lines and 34 triangles (b) 9 straight lines and 34 triangles
(c) 9 straight lines and 36 triangles (d) 10 straight lines and 36 triangles

03. How many triangles and squares are there in the following figure?

(a) 28 triangles, 5 squares (b) 24 triangles, 4 squares


(c) 28 triangles, 4 squares (d) 24 triangles, 5 squares

04. Count the number of triangles and squares in the following figure:

(a) 28 triangles, 10 squares
(b) 28 triangles, 8 squares
(c) 32 triangles, 10 squares
(d) 32 triangles, 8 squares

05. Count the number of triangles in the following figure:


(a) 15
(b) 16
(c) 17
(d) 18

32 Numerical Ability

06. How many triangles are there in the figure below?


(a) 5
(b) 6
(c) 8
(d) 10

07. Count the number of triangles in the following figure:


(a) 27
(b) 25
(c) 23
(d) 21

08. How many triangles does the following figure have ?


(a) 36
(b) 40
(c) 44
(d) 48

09. A set of 4 parallel lines intersect with another set of 5 parallel lines. How many parallelograms are formed?
(GATE - 18)
(a) 20 (b) 48
(c) 60 (d) 72

10. In a 2 x 4 rectangle grid shown below, each cell is a rectangle. How many rectangles can be observed in the grid?
(EE – 2016) (2M)

(A) 21 (B) 27 (C) 30 (D) 36

KEY for Analytical Figures

01. (b) 02.(c) 03. (a) 04. (c) 05. (c) 06. (d) 07. (a) 08. (d) 09. (c) 10. (c)
33 Numerical Reasoning

2.10 Logical Reasoning

Directions: In each question below are given two statements followed by two conclusions numbered I and II. You
have to take the given two statements to be true even if they seem to be at variance from commonly known facts. Read
the conclusions and then decide which of the given conclusions logically follow from the two given statements, dis-
regarding commonly known facts.
Give answer (a) if only conclusion I follows: (b) if only conclusion II follows; (c) if either I or II follows; (d) if neither
I nor II follows and (e) If both I and II follow.

01. Statements: All planets are moons.


All moons are stars
Conclusions:
I. All moons are planets
II. All planets are stars.

02. Statements: All apples are oranges.


Some oranges are papayas.
Conclusions:
I. Some apples are papayas.
II. Some papayas are apples.

03. Statements: Some players are singers.


All singers are tall.
Conclusions:
I. Some players are tall.
II. All players are tall.

04. Statements: All cars are tables


Some children are tables
Conclusions:
I. Some cars are children.
II. Some children are cars

05. Statements: Some fools are intelligent


Some intelligent are great
Conclusions:
I. Some fools are great.
II. All great are intelligent.
34 Numerical Ability

06. Statements: Some bottles are pencils.


Some pencils are glasses.
Conclusions:
I. No glass is bottle.
II. Some bottles are glasses.

07. Statements: All boys are honest.


Sachin is honest
Conclusions:
I. Sachin is a boy
II. All honest persons are boys.

08. Statements: Lawyers married only fair girls.


Shobha is very fair
Conclusions:
I. Shobha was married to a lawyer.
II. Shobha was not married to a lawyer.

09. Statements: All windows are doors.


No door is wall.
Conclusions:
I. No window is wall.
II. No wall is door.

10. Statements: All locks are keys.


No key is a spoon.
Conclusions:
I. No lock is a spoon.
II. No spoon is a lock.

11. Some students were not involved in the strike.


If the above statement is true, which of the following conclusions is/are logically necessary?
1. Some who were involved in the strike were students.
2. No students was involved in the strike.
3. At least one student was involved in the strike.
4. Some who were not involved in the strike were students.
(GATE - 2019)
(a) 1 and 2 (b) 3
(c) 4 (d) 2 and 3
35 Numerical Reasoning

12. Consider the following three statements:


(i) Some roses are red
(ii) All red flowers fade quickly
(iii) Some roses fade quickly
Which of the following statements can be logically inferred from the above statements?
(a) if (i) is true and (ii) is false, then (iii) is false
(b) if (i) is true and (ii) is false, then (iii) is true
(c) if (i) and (ii) are true, then (iii) is true
(d) if (i) and (ii) are false, then (iii) is false (GATE - 2018)

13. Consider the following sentences.


All benches are beds. No bed is a bulb. Some bulbs are lamps.
Which of the following can be inferred ? (CE-2017)
i. some beds are lamps ii. some lamps are beds
(a) Only i (b) Only ii
(c) Both i and ii (d) Neither i nor ii

14. Some tables are shelves. Some shelves are chairs. All chairs are benches. Which of the following conclusions can
be deduced from the preceding sentences? (EC-2017)
(i) At least one bench is a table
(ii) At least one shelf is a bench
(iii) At least one chair is a table
(iv) All benches are chairs
(a) Only i (b) Only ii
(c) Only ii and iii (d) Only iv

15.
Fact 1: Humans are mammals.
Fact 2: Some humans are engineers.
Fact 3: Engineers build houses
If the above statements are facts, which of the following can be logically inferred?
I. All mammals build houses.
II. Engineers are mammals.
III. Some humans are not engineers. (CE – 2016) (2M)
(a) II only (b) III only
(c) I, II and III (d) I only

KEY for Logical Reasoning

01. (b) 02. (d) 03. (a) 04. (d) 05. (d) 06. (d) 07. (d) 08. (c) 09. (e) 10. (e)
11. (c) 12. (c) 13. (d) 14. (b) 15. (b)
36 Numerical Ability

2.11 Verbal Test of Reasoning

2.1 1(a) Series:


(Directions for questions 1 to 16):

What should in place of the question mark (?) in the following number series.

01. 3 13 33 ? 153 313 633


(a) 65 (b) 73 (c) 63 (d) 75

02. 113 ? 83 68 53 38
(a) 95 (b) 98 (c) 93 (d) 88

03. 52 51 48 43 ? 27 16
(a) 39 (b) 36 (c) 38 (d) 37

04. 8 ? 21 32 46 63 83
(a) 10 (b) 11 (c) 12 (d) 13

05. 1 2 6 ? 31 56 92
(a) 15 (b) 13 (c) 16 (d) 18

06. 789 645 545 481 ? 429 425


(a) 440 (b) 441 (c) 444 (d) 445

07. 4 9 19 39 79 ? 319
(a) 157 (b) 160 (c) 159 (d) 158

08. 15 33 104 ? 2124 12755 89298
(a) 424 (b) 420 (c) 421 (d) 423

09. ? 15 49 201 1011 6073


(a) 5 (b) 6 (c) 8 (d) 7

10. 8 36 152 620 2496 10004 ?


(a) 8190 (b) 8187 (c) 40040 (d) 8163

11. 4 10 33 136 685 ?


(a) 3430 (b) 4802 (c) 5145 (d) 4116
37 Numerical Reasoning

12. The missing number in the given sequence 343, 1331, _______ 4913 is (GATE - 19)
(a) 2744 (b) 2197
(c) 3375 (d) 4096

13. What is the missing number in the following sequence?


2, 12, 60, 240, 720, 1440, ______, 0 (GATE - 18)
(a) 2880 (b) 1440
(c) 720 (d) 0

14. The next term in the series 81, 54, 36, 24, ….. is (GATE – 14)

15. What is the next number in the following series (GATE – 14)
12 35 81 173 357 ?

16. Fill in the missing number in the series (GATE – 14)


2 3 6 15 __ 157.5 630

(Directions for questions 17 to 19):


In each of the following questions, a number series is given. After the series, below it another number series is
given, followed by (a), (b), (c), (d) and (e). You have to complete the series starting with the number given, ac-
cording to the sequence of the given series.

17. 36 144 240 324 396 456


59 (a) (b) (c) (d) (e)
Which of the following will come in place of (e)?
(a) 479 (b) 453
(c) 464 (d) 429

18. 123 187 312 528 871 1383


231 (a) (b) (c) (d) (e)
Which of the following will come in place of (d)?
(a) 900 (b) 979
(c) 969 (d) 927

19. 3 3 6 18 72 360 2160


9 (a) (b) (c) (d) (e) (f)
Which of the following will come in place of (f)?
(a) 6225 (b) 6250
(c) 6480 (d) 6280
38 Numerical Ability

(Directions for questions 20 to 25):


Find the missing term in the following sequences

20. AD, CG, FK, JP, ____ (GATE – 12)


(a) PV (b) PW
(c) OV (d) OW

21. 7G, 11K, 13M, ____ (GATE – 14)


(a) 15Q (b) 17Q
(c) 15P (d) 17P

22. 13M, 17Q, 19S, ____ (GATE – 14)


(a) 21W (b) 21V
(c) 23W (d) 23V

23. A, CD, GHI, ____, UVWXY (GATE – 15)


(a) LMN (b) MNO
(c) MNOP (d) NOPQ

24. Find the missing sequence in the letter series B,FH,LNP,------ (ME – 2016) (2M)
(a) SUWY (b) TUVW
(c) TVXZ (d) TWXZ

25. Find the missing group of letters in the following series:


BC, FGH, LMNO, _______ (GATE - 2018)
(a) UVWXY (b) TUVWX
(c) STUVW (d) RSTUV

(Directions for questions 26 to 30):


Find the missing terms in each of the following questions:

26. a_c_b_a_ccb_
(a) bcabc (b) abccb (c) bcaba (d) bbaca

27. –baa–aab–a–a
(a) aaba (b) abab (c) baab (d) aabb

28. a–ba–bb–ab–a
(a) aaba (b) baab (c) baaa (d) abab
39 Numerical Reasoning

29. abc–d–bc–d–b–cda
(a) bacde (b)cdabe (c) dacab (d) decdb

30. PQR–PQ–PP–RPP–RP
(a) QPPP (b) PPQQ (c) PRQQ (d) QQPP

KEY for 2.11(a)

01. (b) 02.(b) 03. (b) 04. (d) 05. (a) 06. (d) 07. (c) 08. (d) 09. (b) 10. (c)
11. (d) 12. (b) 13. (b) 14. 16 15. 725 16. 45 17. (a) 18. (b) 19. (c) 20. (c)
21. (b) 22. (c) 23. (c) 24. (c) 25. (b) 26. (c) 27. (b) 28. (b) 29. (c) 30. (c)

2.11 (b) Classification:


01. Pick the odd one out in the following:
13, 23, 33, 43, 53 (EE – 16) (1M)
(a) 23 (b) 33 (c) 43 (d) 53

02. The number that least fits this set: (324, 441, 97 and 64) is ____________.
(EC/IN – 16) (1M)
(a) 324 (b) 441 (c) 97 (d) 64

03. Which number does not belong in the series below? (GATE – 14)
2, 5, 10, 17, 26, 37, 50, 64
(a) 17 (b) 37 (c) 64 (d) 26

(Directions for questions 04 to 07):


Choose the number which is different from the rest in each of the following questions.

04. 37,65,145,197,323
(a) 65 (b) 145 (c) 197 (d) 323

05. 3456,4536,6352,6354
(a) 3456 (b) 6352 (c) 4536 (d) 6354

06. 125,216,525,729
(a) 125 (b) 216 (c) 729 (d) 525

07. 13339, 13452, 13564, 13678, 13791
(a) 13791 (b) 13678 (c) 13339 (d) 13564
40 Numerical Ability

(Directions for questions 08 and 10):


In each of the following questions find out the group of letters which is different from the others

08. Find the odd one in the following group: ALRVX, EPVZB, ITZDF, OYEIK (GATE – 14)
(a) ALRVX (b) EPVZB (c) ITZDF (d) OYEIK

09. Find the odd one in the following group (GATE – 14)
QWZB BHKM WCGJ MSVX
(a) QWZB (b) BHKM (c) WCGJ (d) MSVX

10. Find the odd one from the following group (GATE – 14)
WEKO IQWA FNTX NVBD
(a) WEKO (b) IQWA (c) FNTX (d) NVBD

KEY for 2.11(b)


01. (b) 02.(c) 03. (c) 04. (d) 05. (b) 06. (d) 07. (d) 08. (d) 09. (c) 10. (d)

2.11 (c) Analogy


(Directions for questions 1 to 07):

In each of the following questions which alternative will replace the question mark:
01. 25:37 :: 49: ?
(a) 41 (b) 65 (c) 56 (d) 60

02. 8:24 :: ?:32
(a) 5 (b) 6 (c) 10 (d) 8

03. 24:60 :: 120: ?
(a) 160 (b) 220 (c) 300 (d) 108

04. 27:125 :: 64: ?
(a) 162 (b) 216 (c) 517 (d) 273

05. 68:130 :: ?: 350
(a) 220 (b) 224 (c) 222 (d) 226

06. 144:10 :: 169: ?


(a) 14 (b) 11 (c) 13 (d) 12

07. MO:1312 :: HJ: ?
(a) 810 (b) 817 (c) 812 (d) 816
41 Numerical Reasoning

(Directions for questions 08 to 10):


In each of the following questions find out the alternative which will replace the question mark.

08. REASON : SFBTPO :: THINK: ?


(a) SGHMJ (b) UIJOL (c) UHNKI (d) UJKPM

09. ? : QEHMDF :: WIDELY : HVCDXK


(a) FRINGE (b) STRING (c) FRANCE (d) DEMAND

10. ACEG : IKMO :: QSUW : ?


(a) YZCE (b) YACD (c) YACE (d) YBCE


KEY for 2.11(c)

01. (b) 02.(b) 03. (c) 04. (b) 05. (c) 06. (b) 07. (b) 08. (b) 09. (a) 10. (c)
Chapter 3 Numerical Computation
3.1 Number System

01. If the number 715 ? 423 is divisible by 3 (? denotes the missing digit in the thousandths place), then the
smallest whole number in the place of ? is __________. (GATE - 2018)
(a) 0 (b) 2 (c) 5 (d) 6

02. A number consists of two digits. The sum of the digits is 9. If 45 is subtracted from the number, its digits are
interchanged. What is the number? (GATE - 2018)
(a) 63 (b) 72 (c) 81 (d) 90

03. What would be the smallest natural number which when divided either by 20 or by 42 or by 76 leaves a remainder
of 7 in each case? (GATE - 2018)
(a) 3047 (b) 6047 (c) 7987 (d) 63847

04. The sum and product of two integers are 26 and 165 respectively. The difference between these two integers is
____. (GATE - 2019)
(a) 3 (b) 6 (c) 2 (d) 4

05. A three –digit number is selected such that it contains no zeros. Now this three-digit number is written beside it
self to form the six-digit number. Its factor is___
(a) 5 (b) 11 (c) 4 (d) None of these

06. A child was asked to add first few natural numbers (that is 1+2+3+……..) so long his patience permitted. As he
stopped, he gave the sum as 575. When the teacher declared the result wrong, the child discovered he had missed
one number in the sequence during addition. The number he missed was:
(a) less than 10 (b) 10 (c) 15 (d) more than 15

07. Anita had to do a multiplication. Instead of taking 35 as one of the multipliers, she took 53. As a result, the product
went up by 540. What is the new product?
(a) 1050 (b) 54 (c) 1440 (d) 1590

08. Three friends, returning from a movie, stopped to eat at a restaurant. After dinner, they paid their bill
and noticed a bowl of mints at the front counter. Sita took 13 rd of the mints, but returned four be-
cause she had a monetary pang of guilt. Fatima then took 14 th of what was left but returned three
for similar reasons. Eswari then took half of the remainder but threw two back into the bowl. The
bowl had only 17 mints left when the raid was over. How many mints were originally in the bowl?
(a) 38 (b) 31 (c) 41 (d) None of these
43 Numerical Computation

09. In a certain shop, 9 oranges cost as much as 5 apples, 5 apples cost as much as 3 mangoes, 4 mangoes cost as much
as 9 lemons. If 3 lemons cost 48 paise, the price of an orange is
(a) 12 paise (b) 14 paise (c)13 paise (d) 15 paise

10. A wholesale tea dealer has 408 kilograms, 468 kilograms and 516 kilograms of three different qualities of tea. He
wants it all to be packed into boxes of equal size without mixing. Find the capacity of the largest possible box.
(a) 50 (b) 36 (c) 24 (d) 12

11. Three different containers contain different quantities of mixture of milk and water whose measurements are 403
kg, 434 kg and 465 kg. What biggest measure must be there to measure all the different quantities exactly?
(a) 1 kg (b) 7 kg (c) 31 kg (d) 41 kg

12. The traffic lights at three different road crossings change after every 48 seconds, 72 seconds and 108 seconds,
respectively. If they all change simultaneously at 8.20: 00 hrs, then they will again change simultaneously at
(a) 8:27:12 h (b) 8:27:24 h (c) 8: 27: 36 h (d) 8: 27: 48 h

13. Three men start together to travel the same way around a circular track of 11 kilometers in circumference. Their
speeds are 4, 5 12 and 8 km/h, respectively. When will they meet at the starting point?
(a) 11 hours (b) 12 hours (c) 23 hours (d) 22 hours

14. The L.C.M of two numbers is 4800 and their H.C.F is 160. If one of the numbers is 480, then the second number
is
(a) 16 (b) 16000 (c)160 (d) 1600

15. A simple mechanical device has four gears A, B, C and D such that they mesh in the order A-B-C-D. The number
of teeth in these gear is 10, 21, 12, 17 respectively. If we start the mechanism from rest, how many revolutions
will the largest gear have to turn before one full cycle is completed and the gears are in the position from where
they started?
(a) 270 (b) 340 (c) 7140 (d) 285

16. Find the greatest possible length of the planks if three pieces of timber 42 m, 49m and 63 m long have to be
divided into planks of the same length?
(a) 8 m (b) 49 m (c) 7 m (d) 63 m

17. If the LCM of two numbers is 750 and their product is 18750. Find the HCF of the numbers.
(a) 40 (b) 35 (c) 25 (d) 45

18. The product of three integers X, Y and Z is 192. Z is equal to 4 and P is equal to the average of X and Y. What is
the minimum possible value of P? (GATE - 2019)
(a) 9.5 (b) 6 (c) 8 (d) 7
44 Numerical Ability

19. Five bells toll at intervals of 5, 6, 10, 12 and 15 seconds respectively. If they toll together at the same time. After
how many seconds will they toll together again
(a) 60 (b) 120 (c) 30 (d)180

20. How many integers are there between 100 and 1000 all of whose digits are even? (GATE - 2019)
(a) 100 (b) 90 (c) 60 (d) 80

KEY for Number System

01. (b) 02. (b) 03. (c) 04. (d) 05. (b) 06. (d) 07. (d) 08. (d) 09. (a) 10. (d)
11. (c) 12. (a) 13. (d) 14. (d) 15. (b) 16. (c) 17. (c) 18. (d) 19. (a) 20. (a)

3.2 Ratio, Proportion and Variation

01. If A : B = 8 : 15, B : C = 5 : 8 and C : D = 4 : 5, then A ; D is equal to :


(a) 2 : 7 (b) 4 : 15 (c) 8 : 15 (d) 15 : 4

02. The ratio of number of males to number of females in a club is 7 : 4. If there are 84 males in the club, the total
number of members in the club is
(a) 126 (b) 132 (c) 136 (d)148

03. The sum of money is divided among P, Q & R, in the ratio of 2 : 3 : 5. If the amount of P and R together is ₹ 400/-
more than that of Q. What is R’s Amount?
(a) ₹ 200 (b) ₹ 300 (c) ₹500 (d) ₹ 400

04. Seats for Mathematics, Physics and Biology in a school are in the ratio 5 : 7 : 8. There is a proposal to increase
these seats by 40%,50% and 75% respectively. What will be the ratio of increased seats .
(a) 2 : 3 : 4 (b) 6 : 7 : 8 (c) 6 : 8 : 9 (d) None of these

05. Salaries of Ravi and Sumit are in the ratio 2:3. If the salary of each is increased by ₹ 4000, the new ratio becomes
40:57. What is Sumit’s present salary ?
(a) ₹ 17,000 (b) ₹20.000 (c) ₹25,500 (d) None of these

2
06. If ₹ 510 be divided among A, B & C in such a way that A gets 3 of what B gets and B gets 14 of what C gets,
then their shares respectively are
(a) ₹ 120,₹ 240,₹.150 (b) ₹60, ₹ 90., ₹ 360 (c) ₹150, ₹ 300, ₹ 60 (d) None of these
45 Numerical Computation

07. An amount of ₹ 2430 is divided among A, B and C such that if their shares be reduced by ₹5, ₹10 and Rs.15
respectively, the remainders shall be in the ratio of 3:4:5. Then, B’s share was :
(a) ₹ 605 (b) ₹ 790 (c) ₹ 800 (d) ₹ 810

08. An amount of ₹ 735 was divided between A, B, C. If each of them had received ₹ 25/- less the shares would have
been in the ratio of 1 : 3 : 2. The money received by C was
(a) ₹ 355 (b) ₹ 135 (c) ₹ 245 (d) ₹ 255

09. In a mixture of 60 litres, the ratio of milk and water is 2 : 1. if this ratio is to be 1 : 2, then the quantity of water to
be further added is :
(a) 20 litres (b) 30 litres (c) 40 litres (d) 60 litres

10. 20 litres of a mixture contains milk and water in the ratio 5 : 3. If 4 litres of this mixture is replaced by 4 litres of
milk, the ratio of milk to water in the new mixture would be
(a) 2 : 1 (b) 7 : 3 (c) 8 : 3 (d) 4 : 3

11. 60 kg of an alloy A is mixed with 100 kg of alloy B. If alloy A has lead and tin in the ratio 3 : 2 and alloy B has
tin and copper in the ratio 1 : 4, then the amount of tin in the new alloy is :
(a) 36 kg (b) 44 Kg (c) 53 kg (d) 80 kg

12. A and B are two alloys of gold and copper prepared by mixing metals in the ratio 7 : 2 and 7 : 11 respectively. If
equal quantities of the alloys are melted to form a third alloy C. the ratio of gold and copper in C will be :
(a) 5 : 7 (b) 5 : 9 (c) 7 : 5 (d) 9 : 5

13. Two alloys A and B contain gold and copper in the ratios of 2:3 and 3:7 by mass, respectively. Equal masses of
alloys A and B are melted to make an alloy C. The ratio of gold to copper in alloy C is _________.(GATE- 2018)
(a) 5:10 (b) 7:13 (c) 6:11 (d) 9:13

14. The ratio of the number of boys and girls who participated in an examination is 4 : 3. The total percentage of
candidates who passed the examination is 80 and the percentage of a girls who passed is 90. The percentage of
boys who passed is _________. (GATE - 2019)
(a) 90.00 (b) 80.50 (c) 55.50 (d) 72.50

15. In a bag, there are coins of 25 p, 10p and 5 p in the ratio of 1 : 2 :3. If there are ₹ 30 in all, how many 5 p coins
are there ?
(a) 50 (b) 100 (c) 150 (d) 200

KEY for Ratio, Proportion and Variation

01. (b) 02. (b) 03. (c) 04. (a) 05. (d) 06. (b) 07. (d) 08.(c) 09. (d) 10. (b)
11. (b) 12. (c) 13. (b) 14. (d) 15. (c)
46 Numerical Ability

3.3 Partnership

01. Yogesh started a business, investing ₹45,000/- After 3 months pranab joined him with a capital of ₹.60,000/-.
After another 6 months, Ashok joined with a capital of ₹.90,000/- At the end of the year they made a profit of
₹20,000/-. What would be Ashok share in it
(a) ₹.8000 (b) ₹.4000 (c) ₹. 3000 (d) ₹. 2000

02. A, B and C enter into a partnership. They invest ₹.40,000. ₹.80,000 and ₹.1,20,000 respectively. At the end of the
first year, B withdraws ₹.40,000, while at the end of the second year, C withdraws ₹.80,000. In what ratio will the
profit be shared at the end of 3 years?
(a) 2 : 3 : 5 (b) 3 : 4 : 7 (c) 4 : 5 : 9 (d) None of these

03. Shekhar started a business investing ₹.25,000 in 1999. In 2000, he invested an additional amount of ₹.10,000 and
Rajeev joined him with an amont of ₹.35,000. In 2001, Shekhar invested another additional amount of ₹.10,000
and Jatin joined them with an amount of ₹.35,000. What will be Rajeev’s share in the profit of ₹.1,50,000 earned
at the end of 3 years from the start of the business in 1999 ?
(a) ₹.45,000 (b) ₹.50,000 (c) ₹.70,000 (d) ₹.75,000

04. A and B entered into a partnership investing ₹.16,000 and ₹.12,000 respectively. After 3 months A withdraw
₹.5000 while B invested ₹.5000 more. After 3 more months C joins the business with a capital of ₹.21,000. The
share of B exceeds that of C, out of a total profit of ₹.26,400 after one year by :
(a)₹.2400 (b) ₹.3000 (c) ₹. 3600 (d) ₹.4800

05. Three partners A, B, C, start a business. Twice A’s capital is equal to thrice B’s capital and B’s capital is four times
C’s capital. Out of a total profit is ₹.16,500 at the end of the year, B’s share is :
(a) ₹. 4000 (b) ₹.6000 (c) ₹. 7500 (d) ₹. 6600

06. A, B and C enter into partnership A invests some money at the beginning, B invests double the amount of after 6
months and C invests thrice the amount after 8 months. If the annual profit be ₹. 27,000, C’s share is :
(a) ₹. 8625 (b) ₹. 9000 (c) ₹.10,800 (d) ₹.11,250

07. A and B started a business jointly. A’s investment was thrice the investment of B and the period of his investment
was two times the period of investment of B. If B received ₹.4000 as profit, then their total profit is :
(a) ₹.16,000 (b) ₹. 20,000 (c) ₹. 24,000 (d) ₹. 28,000

08. ‘A’ is a working partner and ‘B’ is a sleeping partner in a business. A invests ₹.12000/-, and B invests ₹.20,000/-.
A receives 10% of the profit for managing and rest being divided in proportion to their capitals out of a total profit
of ₹.9600/-, The money received by A is
(a) ₹.4200/- (b) ₹. 4600/- (c) ₹. 5100/- (d) ₹. 4800/-
47 Numerical Computation

09. A, B and C jointly thought of engaging themselves in a business venture. It was agreed that A would invest ₹.
6500 for 6 months, B, ₹. 8400 for 5 months, and C, ₹.10,000 for 3 months. A wants to be the working member for
which he was to receive 5% of the profits. The profit earned was ₹.7400. Calculate the share of B in the profit.
(a) ₹.1900 (b) ₹.2660 (c) ₹.2800 (d) ₹.2840

10. A and B started a business with initial investments in the ratio 14 : 15 and their profits were in the ratio 7 : 6. If A
invested the money for 10 months, for how many months did B invests his money
(a) 8 months (b) 4 months (c) 6 months (d) 12 months

KEY for Partnership


01. (b) 02. (b) 03. (b) 04. (c) 05. (b) 06. (b) 07. (d) 08. (a) 09. (b) 10. (a)

3.4 Averages

01. A family consists of grandparents, parents and three grandchildren. The average age of the grand parents is 67
years, that of the parents is 35 years and that of the grandchildren is 6 years. What is the average age of the family?

(a) 28 74 years (b) 31 75 years

(c) 32 17 years (d) None of these

02. A library has an average of 510 visitors on Sundays and 240 on other days. The average number of visitors per
day in a month of 30 days beginning with a Sunday is:
(a) 250 (b) 276 (c) 280 (d) 285

03. Out of 9 persons, 8 persons spent ₹.30 each for their meals. The ninth one spent ₹.20 more than the average
expenditure of all the nine. The total money spent by all of them was:
(a) ₹.260 (b) ₹. 290 (c) ₹. 292.50 (d) ₹. 400.50

04. The batting average for 40 innings of a cricket player is 50 runs. His highest score exceeds his lowest score by
172 runs. If these two innings are excluded, the average of the remaining 38 innings is 48 runs. The highest score
of the player is :
(a) 165 runs (b) 170 runs (c) 172 runs (d) 174 runs
48 Numerical Ability

05. The average temperature of the town in the first four days of a month was 58 degrees. The average for the second,
third, fourth and fifth days was 60 degrees. If the temperatures of the first and fifth days were in the ratio 7 : 8,
then what is the temperature on the fifth day?
(a) 64 degrees (b) 62 degrees (c) 56 degrees (d) None of these

06. The mean temperature of Monday to Wednesday was 370C and of Tuesday to Thursday was 340C. If the temperature
on Thursday was 54 that of Monday, the temperature on Thursday was_______
(a) 380C (b) 360C (c) 400C (d) 390C

07. The average age of class of 36 students is 17 years. When the age of the teacher also included, the average will be
increased by 1 year. What is the age of teacher ?
(a) 50 years (b) 52 years (c) 54 years (d) 56 years

08. The average salary of 35 workers is ₹.1800/-. When the salary of the manager is also included, the average will
be increased by ₹.125/-. What is the salary of manager?
(a) ₹. 6200 (b) ₹.6300 (c) ₹.6400 (d) ₹.6500

09. In a management entrance test, a student scores 2 marks for every correct answer and loses 0.50 marks for
every wrong answer. A student attempts all the 100 questions and scores 120 marks. The number of questions he
answered correctly was
(a) 50 (b) 45 (c) 60 (d) 68

10. The average marks of a students in 10 papers are 80. If the highest and the lowest scores are not considered, the
average is 81. If the highest score is 92, find the lowest?
(a) 55 (c) 60 (c) 62 (d) Cannot be determined

11. The captain of a cricket team of 11 members is 26 years old and the wicket keeper is 3 years older. If the ages of
these two are excluded, the average age of the remaining players is one year less than the average age of the whole
team. What is the average age of the team ?
(a) 23 years (b) 24 years (c) 25 years (d) None of these

12. The average weight of 45 students in a class is 52 kg. Five of them whose average weight is 48 kg leave the class
and other 5 students whose average weight is 54 kg join the class. What is the new average weight (in kg) of the
class?
(a) 52 13 (b) 52 12 (c) 53 32
(d) None of these

13. The average weight of 3 men A,B, and C is 84 kg. Another man D joins the group and the average now becomes
80 kg. If another man E, whose weight is 3 kg more than that of D, replaces A, then the average weight of B,C,D
and E becomes 79 kg. The weight of A is:
(a) 70 kg (b) 72 kg (c) 75 kg (d) 80 kg
49 Numerical Computation

14. In a sequence of 12 consecutive odd numbers, the sum of the first 5 numbers is 425. What is the sum of the last 5
numbers in the sequence? (GATE-14)

15. The sum of eight consecutive odd numbers is 656. the average of four consecutive even numbers is 87. What is
the sum of the smallest odd number and second largest even number (GATE-14)

KEY for Averages

01. (b) 02. (d) 03. (c) 04. (d) 05. (a) 06. (b) 07. (c) 08. (b) 09. (d) 10. (b)
11. (a) 12. (d) 13. (c) 14. 495 15. 163

3.5 Problem on Ages

01. The ratio between the present ages of P and Q is 5 : 7 respectively. If the difference between Q’s present age and
P’s age after 6 years is 2, what is the total of P’s and Q’s present ages ?
(a) 48 years (b) 52 years
(c) Cannot be determined (d) None of these

02. Four years ago, the father’s age was three times the age of his son. The total of the ages of the father and the son
after four years, will be 64 years. What is the father’s age at present?
(a) 32 years (b) 36 years (c) Data inadequate (d) None of these

03. Six years ago, the ratio of the ages of Kunal and Sagar was 6:5. Four years hence, the ratio of their ages will be
11:10. What is Sagar’s age at present?
(a) 16 years (b) 18 years (c) Cannot be determined (d) None of these

04. The ratio between the school age of Neelam and Shaan is 5 : 6 respectively. If the ratio between the one-third age
of Neelam and half of Shaan’s age is 5 : 9, then what is the school age of Shaan ?
(a) 25 years (b) 30 years (c) Cannot be determined (d) None of these

05. The ratio between the present ages of A and B is 5 :3 respectively. The ratio between A’s age 4 years ago and B’s
age 4 years hence is 1 :1. What is the ratio between A’s age 4 years hence and B’s age 4 years ago?
(a) 1 : 3 (b) 2 : 1 (c) 3 : 1 (d) 4 : 1

KEY for Problem on Ages


01. (a) 02. (d) 03. (a) 04. (c) 05. (c)
50 Numerical Ability

3.6 Time and Work

01. A tyre has two punctures. The first puncture alone would have made the tyre flat in 9 minutes and the second
alone would have done it in 6 minutes. If air leaks out at a constant rate, how long does it take both the punctures
together to make it flat?
(a) 1 12 minutes (b) 3 12 minutes (c) 3 53
minutes (d) 4 14 minute

02. P, Q and R are three typists who working simultaneously can type 216 pages in 4 hours. In one hour, R can type
as many pages more than Q and Q can type more than P. During a period of five hours, R can type as many pages
as P can during seven hours. How many pages does each of them type per hour?
(a) 14,17,20 (b) 15,17,22 (c) 15,18,21 (d) 16,18,22

03. P can complete a work in 12 days working 8 hours a day. Q can complete the same work in 8 days working 10
hours a day. If both P and Q work together, working 8 hours a day, in how many days can they complete the work?

5 6 5 6
(a) 5 11 (b) 5 11 (c) 6 11 (d) 6 11

04. A can do a certain work in the same time in which B and C together can do it. If A and B together could do it in
10 days and C alone in 50 days, then B alone could do it in:
(a) 15 days (b) 20 days (c) 25 days (d) 30 days

05. A machine P can print one lakh books in 8 hours, machine Q can print the same number of books in 10 hours while
machine R can print them in 12 hours. All the machines are started at 9 a.m. while machine P is closed at 11 a.m.
and the remaining two machines complete the work. Approximately at what time will the work be finished ?
(a) 11.30 a.m. (b) 12 noon (c) 12.30p.m (d) 1 p.m.

06. A and B can do a piece of work in 30 days, while B and C can do the same work in 24 days and C and A in 20 days.
They all work together for 10 days when B and C leave. How many days more will A take to finish the work?
(a) 18 days (b) 24 days (c) 30 days (d) 36 days

07. A Tailor is able to finish a consignment of Garment fabrication in 80 days less than second Tailor. If the first tailor
is thrice as fast as the second tailor, In how many days both a of them can finish this consignment together
(a) 120 days (b) 40 days (c) 30 days (d) 20 days

08. A alone can do a work in 10 days, which B alone can do it 15 days. If they work together and finish it, then out of
total wages of₹.225/-, the amount ‘A’ will get is
(a)₹135 (b) ₹90 (c) ₹130 (d) ₹120
51 Numerical Computation

09. 5 Skilled workers can build a wall in 20 days, 8 semi-skilled workers can build a wall in 25 days; 10 unskilled
workers can build a wall in 30 days. If a team has 2 skilled 6 semi-skilled and 5 unskilled workers, how long will
it take to build the wall? (ALL BRANCHES-2010)(2M)
(a) 20 days (b) 18 days (c) 16 days (d) 15 days

10. P,Q,R and S are working on a project. Q can finish the task in 25 days, working alone for 12 hours a day. R can
finish the task in 50 days, working alone for 12 hours per day. Q worked 12 hours a day but took sick leave in the
beginning for two days. R worked 18 hours a day on all days. What is the ratio of work done by Q and R after 7
days from the start of the project? (EC/ME – 2016) (2M)
(a) 10:11 (b) 11:10 (c) 20:21 (d) 21:20

11. A and B can complete a piece of work in 9 and 12 days respectively. If they work on alternate days starting with
‘A’, in how many days will the work be completed.
(a) 10 (b) 10 1 (c) 10 1 (d) 10 3
4 2 4

12. A contract is to be completed in 56 days and 104 men were set to work, each working 8 hours a day. After 30 days,
2/5 of the work is finished. How many additional men may be employed so that work may be completed on time
each man now working 9 hours per day?
(a) 46 men (b) 48 men (c) 52 men (d) 56 men

13. Two coal loading machines each working 12 hours per day for 8 days handles 9000 tones of coal with an efficiency
of 90%. While 3 other coal loading machines at an efficiency of 80% set to handle 12,000 tonnes of coal in 6 days.
Find how many hours per day each should work.
(a) 16 hr/day (b) 14 hr/day (c) 15 hr/day (d) None

14. A contract is to be completed in 52 days and 125 identical robots were employed, each operational for 7 hours a
day. After 39 days, five-seventh of the work was completed. How many additional robots would be required to
complete the work on time, if each robot is now operational for 8 hours a day? (GATE - 2018)
(a) 50 (b) 89 (c) 146 (d) 175

15. Seven machines take 7 minutes to make 7 identical toys. At the same rate, how many minutes would it take for
100 machines to make 100 toys? (GATE - 2018)
(a) 1 (b) 7 (c) 100 (d) 700

KEY for Time and Work


01. (c) 02. (c) 03. (a) 04. (c) 05. (d) 06. (a) 07. (c) 08. (a) 09. (d) 10. (c)
11. (b) 12. (d) 13. (a) 14. (*) 15. (b)
52 Numerical Ability

3.7 Pipes and Cisterns

01. A water tank is two-fifth full. Pipe A can fill a tank in 10 minutes and pipe B can empty it in 6 minutes. If both the
pipes are open, how long will it take to empty or fill the tank completely ?
(a) 6 min. to empty (b) 6 min. to fill
(c) 9 min. to empty (d) 9 min. to fill

02. Three pipes A, B and C can fill a tank from empty to full in 30 minutes, 20 minutes and 10 minutes respectively.
When the tank is empty, all the three pipes are opened. A,B and C discharge chemical solutions P, Q and R
respectively. What is the proportion of solution R in the liquid in the tank after 3 minutes ?
5 6 7 8
(a) 11 (b) 11 (c) 11 (d) 11

03. An oil tank can be filled by pipe X in 5 hours and pipe Y in 4 hours, each pump working on its own. When the
oil tank is full and the drainage hole is open, the oil is drained in 20 hours. If initially the tank was empty and
someone started the two pumps together but left the drainage hole open, how many hours will it take for the tank
to be filled? (Assume that the rate of drainage is independent of the Head) (GATE - 2019)
(a) 2.00 (b) 2.50 (c) 4.00 (d) 1.50

04. Two taps A and B can fill a tank in 5 hours and 20 hours respectively. If both the taps are open then due to a
leakage, it took 30 minutes more to fill the tank. If the tank is full, how long will it take for the leakage alone to
empty the tank?
(a) 4 12 hrs (b) 9 hrs (c) 18 hrs (d) 36 hrs

05. Two pipes P and Q can fill a tank in 6 hours and 9 hours respectively, while a third pipe R can empty the tank in
12 hours. Initially, P and R are open for 4 hours. Then P is closed and Q is opened. After 6 more hours R is closed.
The total time taken to fill the tank (in hours) is__ (GATE - 2019)
(a) 14.50 (b) 16.50 (c) 15.50 (d) 13.50

06. A tank is filled by three pipes with uniform flow. The first two pipes operating simultaneously fill the tank in the
same time during which the tank is filled by the third pipe alone. The second pipe fill the tank 5 hours faster than
the first pipe and 4 hours slower than the third pipe. The time required by the first pipe is :
(a) 6 hrs (b) 10 hrs (c) 15 hrs (d) 30 hrs

07. Two pipes A and B can fill a tank in 12 minutes and 15 minutes respectively. If both the pipes are opened
simultaneously, and the Pipe A is closed after 3 minutes, then how much more time will it take to fill the tank by
Pipe B ?
(a) 7 min 15 sec (b) 7 min 45 sec (c) 8 min 5 sec (d) 8 min 15 sec
53 Numerical Computation

08. Two pipes A and B can fill in 15 hours and 20 hours respectively while a third pipe C can empty the full tank in
25 hours. All the three pipes are opened in the beginning. After 10 hours, C is closed. In how much time, will the
tank be full?
(a) 12 hrs (b) 13 hrs (c)16 hrs (d) 18 hrs

09. A booster pump can be used for filling as well as for emptying a tank. The capacity of the tank is
2400 m3.The emptying capacity of the tank is 10m3 per minute higher than its filling capacity and the pump needs
8 minutes lesser to empty the tank than it needs to fill it.What is the filling capacity of the pump ?
(a) 50 m3/min (b) 60 m3/min (c) 72 m3/min (d) None of these

10. Two pipes can fill a tank in 20 and 24 minutes respectively and a waste pipe can empty 3 gallons per minute. All
the three pipes working together can fill the tank in 15 minutes. The capacity of the tank is :
(a) 60 gallons (b) 100 gallons (c) 120 gallons (d) 180 gallons

KEY for Pipes and Cisterns


01. (a) 02. (b) 03. (b) 04. (d) 05. (a) 06. (c) 07. (d) 08. (a) 09. (a) 10. (c)

3.8 Time, Speed and Distance

01. A man in a train notices that he can count 21 telephone posts in one minute. If they are known to be 50 metres
apart, then at what speed is the train traveling?
(a) 55 km/hr (b) 57 km/hr (c) 60 km/hr (d) 63 km/hr

02. A certain distance is covered by a cyclist at a certain speed. If a jogger covers half the distance in double the time,
the ratio of the speed of the jogger to that of the cyclist is
(a) 1 : 2 (b) 2 : 1 (c) 1 : 4 (d) 4 : 1

03. Aruna left for city A from city B at 5.20 a.m. She travelled at the speed of 80 km/hr for 2 hours 15 minutes. After
that the speed was reduced to 60 km/hr. If the distance between two cities is 350 kms, at what time did Aruna reach
city A?
(a) 9.20 a.m (b) 9.25 a.m. (c) 9.35 a.m (d) None of these

04. The average speed of a train in the onward journey is 25% more than that in the return journey. The train halts for
one hour on reaching the destination. The total time taken for the complete to and fro journey is 17 hours, covering
a total distance of 800 km. The speed of the train in the onward journey is
(a) 45 km/hr (b) 47.5 km/hr (c) 52 km/hr (d) 56.25 km/hr
54 Numerical Ability

05. A, B and C are on a trip by a car. A drives during the first hour at an average speed of 50 km/hr. B drives during
the next 2 hours at an average speed of 48 km/hr. C drives for the next 3 hours at an average speed of 52 km/hr.
they reached their destination after exactly 6 hours. Their mean speed was:

(a) 50 km/hr (b) 50 13 km/hr (c) 52 km/hr (d) 52 km/hr

06. A tourist covers half of his journey by train at 60 km/h, half of the remainder by bus at 30 km/h and the rest by
cycle at 10 km/h. Average speed of the tourist during the journey is (ESE-19)
(a) 36 km/h (b) 33 km/h (c) 24 km/h (d) 18 km/h

07. A car covers a distance of 715 km at a constant speed. If the speed of the car would have been 10km/hr more, then
it would have taken 2 hours less to cover the same distance. What is the original speed of the car?
(a) 45 km/hr (b) 50 km/hr (c) 55 km/hr (d) 65 km/hr

08. Two men start together to walk a certain distance, one at 4 km/h and another at 3 km/h. The former arrives half an
hour before the later. Find the distance.
(a) 6 km (b) 9 km (c) 8 km (d) None of these

09. A car starts from A and B travelling 20 km an hour. 1 12 hours later another car starts from A and travelling at
the rate of 30 km an hour reaches B 2 12 hours before the first car. Find the distance from A to B.
(a) 280 km (b) 260 km (c) 240 km (d) none of these

10. A car travels from P to Q at a constant speed. If its speed were increased by 10 km/hr, it would have taken one
hours lesser to cover the distance. It would have taken further 45 minutes lesser if the speed was further increased
by 10 km/hr. What is the distance between the two cities ?
(a) 420 km (b) 540 km (c) 600 km (d) 650 km

11. The distance between two cities A and B is 330 km. A train starts from A at 8 a.m. and travels towards B at 60 km/
hr. Another train starts from B at 9 a.m. and travels towards A at 75 kmphr. At what time do they meet?
(a) 10 a.m (b) 10.30 a.m (c) 11 a.m (d) 11.30 a.m.

12. Two stations A and B are 100 km apart on a straight line. One train starts form A at 7.00 am and travels towards B
at 20 km/h speed. Another train starts from B at 8 am and travels towards A at 25 km/h speed. At what time will
they meet?
(a) 10.30 am (b) 11 am (c) 10 am (d) none of these

13. Two buses start from stations A and B and traveled towards each other a speeds of 50 kmph and 60 kmph
respectively. At the time of their meeting the second bus has found 120 km more than first. The distance between
A and B is
(a) 1320 km (b) 1330 km (c) 1310 km (d) 1300 km
55 Numerical Computation

14. Two cars start at the same time from Mumbai and Pune and proceed towards each other at the rate of 60 km and
40 km per hour, respectively. When they meet, it is found that one car has travelled 20 km more than the other.
Find the distance between Mumbai and Pune.
(a) 150 km (b) 100 km (c) 120 km (d) None of these

15. Two persons A and B are at two places P and Q respectively. A walks at v km/h and B is 2 km/h faster than A,
starting simultaneously from where they stand. If they walk towards each other, they meet in 72 min. If they walk
in the same direction, the faster overtakes the slower in 6 hrs. Find their respective speeds (in km/h).
(a) 3 and 5 (b) 4 and 6 (c) 2 12 and 4 12 (d) 3 12 and 5 12

16. Two trains started at 7AM from the same point. The first train travelled north at a speed of 80k/h and the second
train travelled south at a speed of 100km/h. The time at which they were 540km apart is _______ AM.
(GATE - 19)
(a) 9 (b) 10 (c) 11 (d) 11.30

17. A train is moving at a speed of 132 kmph, if length of the train is 110 m, how long will it take to cross
a railway plot form of 165 m long is
(a) 7.5 sec (b) 7 sec (c) 7.2 sec (d) 8.6 sec

18. A train takes 23 sec to past completely through a station of 272 m long and 19 sec to another station is 200 m long.
The length of the train is
(a) 140 m (b) 142 m (c) 144 m (d) 145 m

19. A train 150 m long is running at 95 km/hr. How much time will it take to pass a man moving in the same direction
at 5 km/hr?
(a) 9S (b) 6S (c) 7S (d) None of these

20. From the time the front of a train enters a platform, it takes 25 seconds for the back of the train to leave the
platform, while travelling at a constant speed of 54 km/h. At the same speed, it takes 14 seconds to pass a man
running at 9 km/h in the same directions as the train. What is the length of the train and that of the platform in
meters, respectively? (GATE - 18)
(a) 210 and 140 (b) 162.5 and 187.5
(c) 245 and 130 (d) 175 and 200

KEY for Time, Speed and Distance

01. (c) 02. (c) 03. (d) 04. (d) 05. (b) 06. (c) 07. (c) 08. (a) 09. (c) 10. (a)
11. (c) 12. (d) 13. (a) 14. (b) 15. (b) 16. (b) 17. (a) 18. (b) 19. (b) 20. (d)
56 Numerical Ability

3.9 Mixture and Alligation

01. The cost of Type 1 rice is ₹.15 per kg and Type 2 rice is ₹.20 per kg. If both Type 1 and Type 2 are mixed in the
rate of 2 : 3, then the price per kg of the mixed variety of rice is:
(a) ₹.18 (b) ₹.18.50 (c) ₹.19 (d) ₹.19.50

02. How many kilograms of sugar costing ₹.9 per kg must be mixed with 27 kg of sugar costing ₹.7 per kg so that
there may be a gain of 10% by selling the mixture at ₹.9.24 per kg?
(a) 36 kg (b) 42 kg (c) 54 kg (d) 63 kg

03. Two vessels A and B contain spirit and water mixed in the ratio 5 : 2 and 7: 6 respectively. Find the ratio in which
these mixture be mixed to obtain a new mixture in vessel C containing spirit and water in the ratio 8 : 5 ?
(a) 4 : 3 (b) 3 : 4 (c) 5 : 6 (d) 7 : 9

04. In an alloy, zinc and copper are in the ratio 1:2. In the second alloy the same elements are in the ratio 2:3, in what
ratio should these two alloys be mixed to form a new alloy in which the two elements are in the ratio 5:8
(a) 3:10 (b) 10:3 (c) 6:7 (d) 7:6

05. A milk vendor has 2 cans of milk. The first contains 25% water and the rest milk. The second contains 50% water.
How much milk should he mix from each of the containers so as to get 12litres of milk such that the ratio of water
to milk is 3 : 5?
(a) 4litres, 8litres (b) 6litres, 6litres (c) 5litres, 7litres (d) 7litres, 5litres

06. One quality of wheat at ₹9.30 per kg mixed with another quality at a certain rate in the ratio 8 : 7. If the mixture
so formed be worth ₹10 per kg, what is the rate per kg of the second quality of wheat ?
(a) ₹10.30 (b) ₹10.60 (c) ₹10.80 (d) ₹11

07. Tea worth ₹.126 per kg and ₹.135 per kg are mixed with a third variety in the ratio 1:1:2. If the mixture is worth
₹.153 per kg, the price of the third variety per kg will be :
(a) ₹.169.50 (b) ₹.170 (c) ₹.175.50 (d) ₹.180

08. A container contains 40 litres of milk. From this container 4 litres of milk was taken out and replaced by water.
This process was repeated further two times.How much milk is now contained by the container ?
(a) 26.34 litres (b) 27.36 litres (c) 28 litres (d) 29.16 litres

09. A container originally contains 10 litres of pure spirit. From this container 1 litre of spirit is replaced with 1 litre
of water, Subsequently, 1 litre of the mixture is again replaced with 1 litre of water and this process is repeated one
more time. How much spirit is now left in the container? (GATE - 11)
(a) 7.58 liters (b) 7.84 liters (c) 7 liters (d) 7.29 liters
57 Numerical Computation

10. 8 litres are drawn from a cask full of wine and is then filled with water. This operation is performed three more
times. The ratio of the quantity of wine now left in cask to that of the water is 16 :65. How much wine did the cask
hold originally?
(a) 18 litres (b) 24 litres (c) 32 litres (d) 42 litres

KEY for Mixture and Alligation


01. (a) 02. (d) 03. (d) 04. (a) 05. (b) 06. (c) 07. (c) 08. (d) 09. (d) 10. (b)

3.10 Percentage

01. In a competitive examination in State A, 6% candidates got selected from the total appeared candidates. State B
had an equal number of candidates appeared and 7% candidates got selected with 80 more candidates got selected
than A. What was the number of candidates appeared from each State?
(a) 7600 (b) 8000 (c) 8400 (d) Data inadequate

02. In a country of 1400 million population, 70% own mobile phones. Among the mobile phone owners, only
294 million access the internet. Among these users, only half buy goods from e-commerce portals. What is the
percentage of these buyers in the country? (GATE - 2019)
(a) 10.50 (b) 14.70 (c) 15.00 (d) 50.00

03. In an election between two candidates, one got 55% of the total valid votes, 20% of the votes were invalid. If the
total number of votes was 7500, the number of valid votes that the other candidate got, was :
(a) 2700 (b) 2900 (c) 3000 (d) 3100

04. 10% of the voters did not cast their vote in an election between two candidates. 10% of the votes polled were
found invalid. The successful candidate got 54% of the valid votes and won by a majority of 1620 votes. The
number of votes enrolled on the voters list was:
(a) 25000 (b) 33000 (c) 35000 (d) 40000

05. In an examination, 5% of the applicants were found ineligible and 85% of the eligible candidates belonged to the
general category. If 4275 eligible candidates belonged to other categories, then how many candidates applied for
the examination.?
(a) 30,000 (b) 35,000 (c) 37,000 (d) None of these

06. Gaurav spends 30% of his monthly income on food articles, 40% of the remaining on conveyance and clothes and
saves 50% of the remaining. If his monthly salary is ₹.18,400, how much money does he save every month?
(a) ₹. 3624 (b) ₹. 3864 (c) ₹.4264 (d) ₹.5888
58 Numerical Ability

07. From the salary of an officer, 10% is deduced as house rent, 15% of the rest he spends on children’s education
and 10% of the balance, he spends on clothes. After this expenditure he is left with ₹.1,377. His salary is______.
(a)₹. 2000 (b) ₹.2,040 (c)₹. 2,100 (d) ₹.2,200

08. Fiscal deficit was 4% of the GDP in 2015 and that increased to 5% in 2016. If the GDP increased by 10% from
2015 to 2016, the percentage increase in the actual fiscal deficit is___ (GATE - 2019)
(a) 37.50 (b) 25.00 (c) 35.70 (d) 10.00

09. Mr. X a businessman had the income in the year 2000, such that he earned a profit of 20% on his investment in the
business. In the year 2001, his investment was less by ₹.5000 but still had the same income (income = Investment
+ Profit) as that in 2000. Thus the percent profit earned in 2001 increased by 6%. What was his investment in
2000?
(a) ₹.1,02,000 (b) ₹.1,05,000 (c) ₹.1,50,000 (d) Data inadequate

10. The population of a town is 10,000. It increases by 10% during the first year, During the second year it decreases
by 20% and increased by 30%, during the Third year. What is the population of 3 years will be
(a) 14,440 (b) 12,440 (c) 11,440 (d) 11,240

11. In an examination, 30% and 35% students respectively failed in History and Geography while 27% students failed
in both the subjects. If the number of students passing the examination is 248, find the total number of students
who appeared in the examination.
(a) 425 (b) 380 (c) 400 (d) none of these

12. Three papers where set in an examination and the maximum marks per paper were in the ratio of 1:2:2 respectively.
If a student obtained 50% in the first paper, 60% in the second , and 65% in the third, what percent did he obtain
overall?
(a) 58.3% (b) 66.66% (c) 33.33 % (d) 60 %

13. A house-owner was having his house painted. He was advised that he would require 25 kg of paint. Allowing for
15% wastage and assuming that the paint is available in 2 kg cans, what would be the cost of paint purchased, if
one can costs ₹. 16?
(a) ₹. 240 (b) ₹.180 (c) ₹. 160 (d) ₹. 360

14. One kg of tea and one kg of sugar together cost ₹. 95. If the price of tea fall by 10% and that of sugar rises by
20%, then the price of one kg of each combined comes to ₹. 90. The original price of tea in ₹. per kg is_____.
(a) ₹. 72 (b) ₹. 55 (c) ₹. 60 (d) ₹. 80

15. A person moving through a tuberculosis prone zone has a 50% probability of becoming infected. However, only
30% of infected people develop the disease. What percentage of people moving through a tuberculosis prone zone
remains infected but does not shows symptoms of disease? (EC/ME – 2016) (2M)
(a) 15 (b) 33 (c) 35 (d) 37
59 Numerical Computation

16. One percent of the people country X are taller than 6 ft. Two percent of the people of country Y are taller than
6ft. There are thrice as many people in country X as in country Y. Taking both countries together, what is the
percentage of people taller than 6 ft? (GATE -14)
(a) 3.0 (b) 2.5 (c) 1.5 (d) 1.25

17. In a party, 60% of the invited guests are male and 40% are female. If 80% of the invited guests attended the party
and if all the invited female guests attended, what would be the ratio of males to females among the attendees in
the party? (GATE - 18)
(a) 2 : 3 (b) 1 : 1 (c) 3 : 2 (c) 2 : 1

18. In an examination Suresh scored 22% of the maximum marks and failed the exam by 52 marks, where as Ramesh
scored 45% of the maximum marks and got 40 marks more than the pass mark. Then find the maximum marks of
the exam?
(a) 300 (b) 200 (c) 400 (d) 500

19. Population of state X increased by x% and the population of state Y increased by y% from 2001 to 2011. Assume
that x is greater than y. Let P be the ratio of the population of state X to state Y in a given year. The percentage
increase in P from 2001 to 2011 is _________. (GATE - 2019)

100 (x - y)
(a) 100 + y (b) x – y

x 100 (x - y)
(c) y (d) 100 + x

20.
On time → x%
upto 15 minutes delayed → 43%
15 - 30 minutes delayed → 17%
30 - 60 minutes delayed → 12%
More than 60 minutes delayed →3%
The chart above describes from a certain airport on a certain day. If 1,200 flights were delayed, how many flights
departed on time?
(a) 350 (b) 300 (c) 250 (d) 400

KEY for Percentage


01. (b) 02. (a) 03. (a) 04. (a) 05. (a) 06. (b) 07. (a) 08. (a) 09. (b) 10. (c)
11. (c) 12. (d) 13. (a) 14. (d) 15. (c) 16. (d) 17. (b) 18. (c) 19. (a) 20. (d)
60 Numerical Ability

3.11 Profit, Loss and Discount

01. Sarams purchased 120 reams of paper at ₹.80 per ream. He spent ₹. 280 on transportation, paid octroi at the rate
of 40 paise per ream and paid ₹.72 to the coolie. If he wants to have a gain of 8%, what must be the selling price
per ream ?
(a) ₹. 86 (b) ₹. 87.48 (c)₹. 89 (d) ₹. 90

02. A manufacturer undertakes to supply 2000 pieces of a particular component at ₹.25 per piece. According to his
estimates, even if 5% fail to pass the quality tests, then he will make a profit of 25%. However, as it turned out,
50% of the components were rejected. What is the loss to the manufacturer ?
(a) ₹.12,000 (b) ₹.13,000 (c) ₹. 14,000 (d) ₹.15,000

03. The percentage profit earned by selling an article for ₹.1920 is equal to the percentage loss incurred by selling the
same article for ₹.1280. At what price should the article be sold to make 25% profit ?
(a) ₹. 2000 (b) ₹.2200 (c) ₹. 2400 (d) Data inadequate

04. By mixing two brands of tea and selling the mixture at the rate of ₹.177 per kg, a shopkeeper makes a profit of
18%. If to every 2 kg one brand costing ₹. 200 per kg 3kg of the other brands is added, then how much per kg
does the other brand cost ?
(a) ₹.110 (b) ₹120 (c) ₹.140 (d) None of these

05. The manufacturer of a certain item can sell all he can produce at the selling price of ₹60 each. It costs him ₹40
in materials and labour to produce each item and he has overhead expenses of ₹3000 per week in order to operate
the plant. The number of units he should produce and sell in order to make a profit of at least ₹1000 per week, is:
(a) 200 (b) 250 (c) 300 (d) 400

06. A bought a radio set and spent ₹110 on its repairs. He then sold it to B at 20% profit, B sold it to C at a loss of
10% and C sold it for ₹1188 at a profit of 10%. What is the amount of which A bought the radio set ?
(a) ₹850 (b) ₹ 890 (c) ₹930 (d) ₹ 950

07. A fruit seller sold a basket of fruits at 12.5% loss. Had he sold it for Rs. 108 more, he would have made a 10%
gain. What is the loss in Rupees incurred by the fruit seller? (GATE - 2018)
(a) 48 (b) 52 (c) 60 (d) 108

08. Garima purchased a briefcase with an additional 10% discount on the reduced price after deducting 20% on the
labelled price. If the labelled price was ₹.1400, at what price did she purchase the briefcase ?
(a) ₹ .980 (b)₹.1008 (c) ₹.1056 (d) ₹.1120
61 Numerical Computation

09. A person divided an amount of Rs. 100,000 into two parts and invested in two different schemes. In one he got
10% profit and in the other he got 12%. If the profit percentages are interchanged with these investments he would
have got Rs.120 less. Find the ratio between his investments in the two schemes. (GATE - 2019)
(a) 47 : 53 (b) 9 : 16 (c) 11 : 14 (d) 37 : 63

10. A shopkeeper purchased 150 identical pieces of calculators at the rate of ₹.250 each. He spent an amount of
₹.2500 on transport and packing. he fixed the labelled price of each calculator at ₹.320. However, he decided to
give a discount of 5% on the labelled price. What is the percentage profit earned by him ?
(a) 14% (b) 15% (c) 16% (d) 20%

KEY for Profit, Loss and Discount

01. (d) 02. (b) 03. (a) 04. (d) 05. (a) 06. (b) 07. (c) 08. (b) 09. (a) 10. (a)

3.12 Simple & Compound Interest

01. ₹.800 becomes ₹.956 in 3 years at a certain rate of simple interest. If the rate of interest is increased by 4%, what
amount will ₹. 800 becomes in 3 years?
(a) ₹.1020.80 (b)₹.1025 (c) ₹.1052 (d) Data inadequate

02. Nitin borrowed some money at the rate of 6% p.a. for the first three years, 9% p.a. for the next five years and 13%
p.a. for the period beyond eight years. If the total interest paid by him at the end of eleven years is ₹. 8160, how
much money did he borrow ?
(a) ₹. 8000 (b) ₹. 10,000 (c)₹.12,000 (d) Data inadequate

03. The simple interest on a sum of money will be ₹.600 after 10 years. If the principal is trebled after 5 years, what
will be the total interest at the end of the tenth year ?
(a) ₹.600 (b) ₹.900 (c) ₹. 1200 (d) ₹.1500

04. An automobile financier claims to be lending money at simple interest, but he includes the interest every six
months for calculating the principal. If he is charging an interest of 10%, the effective rate of interest becomes :
(a) 10% (b) 10.25% (c) 10.5% (d) None of these

05. Peter invested an amount of ₹.12,000 at the rate of 10 p.c.p.a simple interest and another amount at the rate of
20 p.c.p.a simple interest. The total interest earned at the end of one year on the total amount invested become 14
p.c.p.a Find the total amount invested.
(a) ₹. 20,000 (b) ₹. 22,000 (c) ₹. 24,000 (d) ₹. 25,000
62 Numerical Ability

06. An amount of ₹. 1,00,000 is invested in two types of shares. The first yields an interest of 9% p.a. and the second,
11% p.a. If the total interest at the end of one year is 9¾ %, then the amount invested in each share was:
(a) ₹.52,500; ₹.47,500 (b) ₹.62,500; ₹.37,500
(c) ₹.72,500; ₹.27,500 (d) ₹. 82,500;₹.17,500

07. Leila aspires to buy a car worth Rs. 10,00,000 after 5 years. What is the minimum amount in Rupees that she
should deposit now in a bank which offers 10% annual rate of interest, if the interest was compounded annually?
(GATE - 18)
(a) 5,00,000 (b) 6,21,000 (c) 6,66,667 (d) 7,50,000

08. If the compound interest on a sum for 2 years at 12½ % per annum is ₹. 510, the simple interest on the same sum
at the same rate for the same period of time is:
(a) ₹. 400 (b) ₹. 450 (c) ₹. 460 (d) ₹. 480

09. Mr. Dhruva invested money in two schemes A and B offering compound interest @ 8 p.c.p.a and 9
p.c.p.a respectively. If the total amount of interest accrued through two schemes together in two years
was ₹.4818.30 and the total amount invested was ₹.27,000, what was the amount invested in scheme
A?
(a) ₹.12,000 (b) ₹. 13,500 (c) ₹. 15,000 (d) Can’t be determined

10. A man borrows ₹. 12,500 at 20% compound interest. At the end of every year he pays ₹. 2000 as part repayment.
How much does he still owe after three such installments?
(a) ₹. 12,000 (b) ₹.12,864 (c) ₹. 15,600 (d) None of these

KEY for Simple & Compound Interest


01. (c) 02. (a) 03. (c) 04. (b) 05. (a) 06. (b) 07. (b) 08. (d) 09. (a) 10. (d)
Chapter 4 Data Interpretation
4.1 Tabulation

Directions (01 to 05): Study the following table carefully and answer the questions given below it:

Number of Different categories of vehicles sold in the country over the years (in thousands)

Year Heavy Light commercial vehicles Cars Jeeps Two-Wheelers


1990 26 64 232 153 340
1991 45 60 242 172 336
1992 72 79 248 210 404
1993 81 93 280 241 411
1994 107 112 266 235 442
Total 331 408 1268 1011 1933

01. The percentage increase in the sales in 1993 over the previous year was maximum for which of the following
categories of vehicles?
(a) Cars (b) Heavy Vehicles
(c) Jeeps (d) Light commercial vehicles

02. In which year was the number of 2-wheelers sold as a percentage of the total number of Vehicles sold during that
year, the highest?
(a) 1994 (b) 1991
(c) 1990 (d) 1992

03. The number of Heavy Vehicles sold in 1993 was approximately what percent of the total number of Vehicles sold
in 1992?
(a) 8 (b) 10 (c) 7 (d) 9

04. If the same percentage increase in the number of Heavy Vehicle as in 1994 over 1993 is expected in 1995,
approximately how many heavy vehicles will be sold in 1995?
(a) 139 (b) 141 (c) 144 (d) 133

05. In which of the following years was the number of light commercial vehicles sold approximately 25% of the
number of 2-wheelers sold?
(a) 1993 (b) 1991 (c) 1990 (d) 1994
64 Numerical Ability

4.2 Bar Charts

Directions (Questions 06 to 08): The following is a multiple bar chart showing men’s and women’s average daily
earnings in certain industries. Study the chart and answer the questions given below.

Multiple Bar chart showing men’s and women’s Average Daily Earnings (in Rs)

= Men (21 years and above)


80
= Women (18 years and above)
70
66.90
Average daily Earnings (Rs.)

60 59.50

50 48.60

40.90 40.60
40
35.80
34.20
30 27.00
21.10
20 18.30

10

2000 2001 2002 2003 2004


Year

06. In which year is the ratio of men’s average daily earnings and women’s average daily earnings, the highest ?
(a) 2000 (b) 2002
(c) 2001 (d) 2004

07. In which year is the percentage increase in the average daily earnings of men over the preceding year, the maximum
?
(a) 2001 (b) 2004
(c) 2002 (d) 2003

08. The difference between the average daily earnings of men and women over successive years _______
(a) increase (b) decrease
(c) remains the same (d) None of these
65 Data Interpretation

Directions (Questions 09 to 13) : Study the bar chart carefully and answer the following questions.

Production of cloth in India


(In million metres)

3389
3356 Mills
3432
3487
5813
5765
5445 Powerloom
5315

3302
3245
3137 Handloom
2956

2003-04 2004-05 2005-06 2006-07

09. Total cloth production in India in the year 2003 – 04 was (in million metres)
(a) 11,758 (b) 12,014 (c) 11,000 (d) 12,504

10. What is the growth percent in the production of cloth in the powerloom sector in 2006 – 07 over 2004 – 05?
(a) 10 (b) 8 (c) 5 (d) 6.8

11. What is the approximate ratio of the production of cloth in the powerloom sector to that in the Handloom sector
during 2003 – 04?
(a) 18 : 13 (b) 9 : 5 (c) 5 : 9 (d) 8 : 3

12. If the average price of a metre of cloth produced in the Handloom, power loom and mill sectors is 2.5 : 1.5 : 1, then
the percentage growth of the value of the cloth produced in 2004 – 05 over 2003 – 2004 was (assume no change
in the relative price)
(a) 12 (b) 3 (c) 16 (d) cannot be determined

13. In the year 2006 – 07, it was found that per capita production of labour employed in Handloom, powerloom and
mill sectors was 1500 metres, 3000 metres, and 5600 metres respectively. Approximately how many people were
employed in the cloth industry durig that year ?
(a) 37 lakhs (b) 5 lakhs (c) 12.5 lakhs (d) 47 lakhs
66 Numerical Ability

4.3 Pie Charts

Directions (Questions 14 to 18): Study the following chart and table carefully and answer the question given
below it:

PROPORTION OF POPULATION OF SEVEN VILLAGES IN 2007
YY
% Population below 15% ZZ
Village 15%
11%
11%
poverty line
X 38
XX
Y 52 16%
16% R
R
Z 42 16%
16%
R 51
S 49
VV
T 46 10%
10% SS
V 58 11%
11%

TT
21%
21%

14. Find the population of village ‘S’ if the population of village X below poverty line in 2007 is 12160?
(a) 18500 (b) 20500 (c) 22000 (d) 26000

15. The ratio of population of village T below poverty line to that of village Z below poverty line in 2007 is _______.
(a) 11 : 23 (b) 13 : 11 (c) 23 : 11 (d) 11 : 13

16. If the population of village R in 2007 is 32000, the what will be the population of village Y below poverty line in
that year?
(a) 14100 (b) 15600 (c) 16500 (d) 17000

17. If in 2008, the population of village Y and V increase by 10% each and the percentage of population below
poverty line remains unchanged for all the villages, then find the population of village V below poverty line in
2008, given that the population of village Y in 2007 was 30,000?
(a) 11250 (b) 12760 (c) 13140 (d) 13780

18. If in 2009, the population of village ‘R’ increases by 10%, while that of village Z reduces by 5% compared to that
in 2007 and the percentage of population below poverty line remains unchanged for all the villages, then find the
approximate ratio of population of village ‘R’ below poverty line to the ratio of population of village ‘Z’ below
poverty line for the year 2009?
(a) 2 : 1 (b) 3 : 2 (c) 4 : 3 (d) 5 : 4
67 Data Interpretation


4.4 Line Graphs / X – Y Charts

Directions (Question 19 to 23): Study the following graph carefully and answer the question given below it:
Metal A Metal B

35
Consumption in 1000 tones

30
25
20
15
10
5
0
1988 1989 1990 1991 1992 1993
Years

19. In how many years, the consumption of metal A was less than the average consumption of metal B in the given
years?
(a) One (b) Two (c) Three (d) Four

20. What was the difference is the consumption of metal A and metal B in 1992?
(a) 500 tonnes (b) 50 tonnes
(c) 5000 tonnes (d) 5 tones

21. In which of the following pairs of years, the consumption of metal B in 1993 was equivalent to the consumption
of metal A?
(a) 1988 and 1989 (b) 1989 and 1993
(c) 1988 and 1993 (d) 1990 and 1992

22. In which of the following year, the consumption of both the metals together was maximum?
(a) 1988 (b) 1990 (c) 1993 (d) 1991

23. What is the per cent decrease in the consumption of metal A in 1992 in comparison to 1991?
(a) 30 (b) 70 (c) 40 (d) 35

24. The bar graph in Panel(a) shows the propagation of male and female illiterates in 2001 and 2011. The proportions
of males and females in 2001 and 2011 are given in Panel(b) and (c), respectively. The total population did not
change during this period.
The percentage increase in the total number of literates from 2001 to 2011 is _____. (GATE-2019)
68 Numerical Ability

Proportion of illiterates (%)


2001 2011
100
80
60
60 50 Female
40 40 Female
40 Male 40% Male 50%
20 60% 50%
0
Female Male

Panel (a) Panel (b) Panel (c)

(a) 35.43 (b) 30.43 (c) 34.43 (d) 33.43

25. The graph given here shows a car following the linear path with uniform speed. Study the graph and answer the
question.
30 (5,30)

24 (4,24)
Distance (in km)

18 (3,18)

12 (2,12)
(1,6)
6

1 2 3 4 5
Time (in hours)

The Speed of the car is


(a) 12 km/hr (b) 6 km/hr (c) 18 km/hr (d) 24 km/hr

26. Study the following table carefully to answer the question that follow:
Percentage of Marks Obtained by Six Students in Five Different Subjects in A school Examination.

Subject Student English (50) Maths (100) Science (150) Hindi(50) Social Studies (75)
P 66 89 80 78 84
Q 58 79 64 82 60
R 62 77 74 84 88
S 72 67 84 74 68
T 70 81 70 76 64
U 64 83 60 88 70

Note: Figures in brackets indicate maximum marks for each subject.


What is the approximate overall percentage of marks obtained by U in all the subjects together?
(a) 75 (b) 71 (c) 79 (d) 82
69 Data Interpretation

27. Mola is a digital platform for taxis in a city. It offers three types of rides- Pool, Mini and Prime. The table below
presents the number of rides for the past four months. The platform earns one US dollar per ride. What is the
percentage share of revenue contributed by Prime to the total revenues of Mola, for the entire duration?
(GATE-2019)
Month
Type
January February March April
Pool 170 320 215 190
Mini 110 220 180 70
Prime 75 180 120 90

(a) 38.74 (b) 25.86 (c) 23.97 (d) 16.24

28. Study the following pie-charts carefully to answer the question that follow:
Percentage break-up of number of children in five different villages and breakup of children attending school from
those villages

Total number of Children =2040 Total Children attending School=1450


M, 32%
M,32%
L,15%
L, 15% M,25%
M, 25%

L,14%
L, 14%

N,12%
N, 12%
N,10%
N, 10%
P, 30%
P,30%
P,22%
P, 22%
O,20%
O, 20% O, 20%
O,20%

What is the respective ratio of total number of children from village O to the number of children attending school
from the same village?
(a) 204 : 145 (b) 179 : 131
(c) 167 : 111 (d) 266 : 137

29. The following graph represents the installed capacity for cement production (in tonnes) and the actual production
(in tonnes) of nine cement plants of a cement company. Capacity utilization of a plant is defined as ratio of actual
production of cement to installed capacity. A plant with installed capacity of at least 200 tonnes is called a large
plant and a plant with lesser capacity is called a small plant. The difference between total production of large
plants and small plants, in tonnes is _______.
(EE – 2016) (2M)


70 Numerical Ability

Installed capacity Actual Production

300
250
Capacity Production (tonnes) 250 230
220
190 200 190 200 190
200 180
160 150 160 160 150
150 140
120 120
100
100

50

0
1 2 3 4 5 6 7 8 9
Plant Number

30. The velocity V of a vehicle along a straight line is measured in m/s and plotted as shown with respect to time in
seconds. At the end of the 7 seconds, how much will the odometer reading increase by (in m)?
(EC/IN – 2016) (2M)

V (m/s)

1 2 3 4 5 6 7 Time (s)

–1


(a) 0 (b) 3 (c) 4 (d) 5

31. A firm hires employees at five different skill levels P, Q, R, S, T. The shares of employment at these skill levels of
total employment in 2010 is given in the pie chart as shown. There were a total of 600 employees in 2010 and the
total employment increased by 15% from 2010 to 2016. The total employment at skill levels P, Q and R remained
unchanged during this period. If the employment at skill level S increased by 40% from 2010 to 2016, how many
employees were there at skill level T in 2016?
(GATE-2019)
71 Data Interpretation

Percentage share of skills in 2010



(a) 35 (b) 60 (c) 72 (d) 30

32. Ghosh Babu was reviewing the performance of his company (see chart).

800
700
600 Revenue
Rupees

500 Variable Cost


400
300 Fixed Cost
200
100
60
0
5 10 15 20 25 30 35 40 45 50
Units

Profit is defined as follows : Profit = Revenue – Variable Cost – Fixed Cost. If the production has to be increased
beyond 34 units, there is a need to increase the fixed assets and the fixed costs will increase. In any case, it is not
possible to increase the production beyond 50 units. Ghosh Babu does not incur loss if the production is
(a) 20 units (b) 10 units
(c) 5 units (d) 0 units

33. The annual average rainfall in a tropical city is 1000 mm. On a particular rainy day (24-hour period), the cumulative
rainfall experienced by the city is shown in the graph. Over the 24-hour period, 50% of the rainfall falling on a
rooftop, which had an obstruction free area of 50 m2, was harvested into a tank. What is the total volume of water
collected in the tank in liters?
(GATE - 18)
72 Numerical Ability

400

350

300

Cumulative rainfall (mm)


250

200

150

100

50

0 3 6 9 12 15 18 21 24
Hours

(a) 25,000 (b) 18,750 (c) 7,500 (d) 3,125

34. A shaving set company sells 4 different types of razors, Elegance, Smooth, Soft and Executive. Elegance sells at
₹. 48, Smooth at ₹. 63, Soft at ₹. 78 and Executive at Rs. 173 per piece. The table below shows the numbers of
each razor sold in each quarter of a year. (CE – 2016) (2M)
Quarter\Product Elegance Smooth Soft Executive
Q1 27300 20009 17602 9999
Q2 25222 19392 18445 8942
Q3 28976 22429 19544 10234
Q4 21012 18229 16595 10109
Which product contributes the greatest fraction to the revenue of the company in that year?
(a) Elegance (b) Executive
(c) Smooth (d) Soft

35. Study the following chart to answer


%Population the question given below:
below Poverty

Proportion of Population of Seven Villages in 1995 Villages %Population below Poverty


A 45
G A B 52
15% 13%
C 38
B
F
16%
D 58
13%
E 46
E
C F 49
8%
18% D G 51
17%
73 Data Interpretation

In 1996, the population villages A as well as B are increased by 10% from the year 1995. If the population of
village A in 1995 was 5,000 and the percentage of population below poverty line in 1996 remain same as in 1995;
find approximately the population of village B below poverty line in 1996.
(a) 4,000 (b) 4,500 (c) 2,500 (d) 3,500

36. Two finance companies, P and Q, declared fixed annual rates of interest on the amounts invested with them. The
rates of interest offered by these companies may differ from year to year. Year-wise annual rates of interest offered
by these companies are shown by the line graph provided below.
9
(CE – 2016) (2M)
9 9.5 10
7 8 P Q
8 8 8
6.5 7.5 6
6.5
4

2000 2001 2002 2003 2004 2005 2006

If the amounts invested in the companies, P and Q, in 2006 are in the ratio 8:9, then the amounts received after
one year as interests from companies P and Q would be in the ratio:
(a) 2:3 (b) 3:4 (c) 6:7 (d) 4:3

37. The bar graph pertains to the estimates of incidence of poverty in India, given as percentage. The table following
the graph gives the population of India during 1971 – 2001 as per census :

Poverty Ratio (% )
56.4

54.9

60
45.7
49

44.5

50
40.8

37.3
Percentage (%)

32.4

40
36

27.3

26.1
23.6

30
20
10
0
1973-74 1983-84 1993-94 1999-00

Rural Urban Comined

Population 1971 1981 1991 2001


All India 548160 684329 846302 1027015

The total number of the poor in 2000 is approximately


(a) 268 millions (b) 278 millions
(c) 242 millions (d) 300 millions
74 Numerical Ability

38. A contour line joins locations having the same height above the mean sea level. The following is a contour plot of
a geographical region. Contour lines are shown at 25 m intervals in this plot.
(EC-2017)
425 Q
550
450
575

P
550 500

500 475

The path from P to Q is best described by


(a) Up-Down-Up-Down (b) Down-Up-Down-Up (c) Down-Up-Down (d) Up-Down-Up

39. A contour line joins locations having the same height above the mean sea level.The following is a contour plot of
a geographical region. Contour lines are shown at 25m intervals in this spot.
(EC-2017) 425
R 550
450 575

Q S
P 575
550 500
T
500 475

0 1 2km

Which of the following is the steepest path leaving from P?


(a) P to Q (b) P to R (c) P to S (d) P to T

40. A contour line joins locations having the same height above the mean sea level. The following is a contour plot of
a geographical region. Contour lines are shown at 25m intervals in this plot. If in a flood the water level rises to
525 m, which of the villages P, Q, R, S, T get submerged? (EE-2017)

425 Q
450 R
550

P
550
T
500
450
500 S

(a) P, Q (b) P, Q, T (c) R, S, T (d) Q, R, S


75 Data Interpretation

41. An air pressure contour line joins locations in a region having the same atmospheric pressure. The following is an
air pressure contour plot of a geographical region. Contour lines are shown at 0.05 bar intervals in this plot
R (EE-2017)
0.65
0.7
0.9
0.95
S 0.95
P 0.9

0.8 0.8
Q

0.75

0 1 2 km

If the possibility of a thunderstorm is given by how fast air pressure rises or drops over a region. Which of the
following regions is most likely to have a thunderstorm?
(a) P (b) Q (c) R (d) S

42. The growth of bacteria (lactobacillus) in milk leads to curd formation. A minimum bacterial population density
of 0.8 (in suitable units) is needed to form curd. In the graph below, the population density of lactobacillus in
1 litre of milk is plotted as a function of time, at two different temperatures, 25oC and 37oC.
(ME-2017)

1.0
0.9 37°C
0.8 25°C
0.7
0.6
Population density

0.5
0.4
0.3
0.2
0.1
0.0

0 20 40 60 80 100 120 140 160 180 200


Time (min)

Consider the following statements based on the data shown above:


(i) The growth in bacterial population stops earlier at 37oC as compared to 25oC
(ii) The time taken for curd formation at 25oC is twice the time taken at 37oC
Which one of the following options is correct?
(a) Only i (b) Only ii
(c) Both i and ii (d) Neither i nor ii
76 Numerical Ability

43. In the graph below, the concentration of a particular pollutant in a lake is plotted over (alternate) days of a month
in winter (average temperature 10oC) and a month in summer (average temperature 30oC).
11
(ME-2017)
10 Winter
9 summer
8
7
Pollutant concentration (ppm)

6
5
4
3
2
1
0

0 2 4 6 8 10 12 14 16 18 20 22 24 26 28 30
Day of the month

Consider the following statements based on the data shown above:


i. Over the given months, the difference between the maximum and the minimum pollutant concentrations is
the same in both winter and summer.
ii. There are at least four days in the summer month such that the pollutant concentrations on those days are
within 1 ppm of the pollutant concentrations on the corresponding days in the winter month.
Which one of the following options is correct?
(a) Only i (b) Only ii
(c) Both i and ii (d) Neither i nor ii

44. The bar graph below shows the output of five carpenters over one month, each of whom made different items of
furniture : chairs, tables, and beds (CE-2017)
Bed
Number of furniture items

Table
20
18 Chair
16
14
12
10
8
6
4
items

2
0
C1 C2 C3 C4 C5
Carpenter (C)
Consider the following statements
(i) The number of beds made by carpenter C2 is exactly the same as the number of tables made by
carpenter C3.
77 Data Interpretation

(ii) The total number of chairs made by all carpenters is less than the total number of tables
Which one of the following is true?
(a) Only i (b) Only ii (c) Both i and ii (d) Neither i nor ii

45. The points in the graph below represent the halts of a lift for durations of 1 minute, over a period of 1 hour.
5
Floor Number
4
3
2
1
0
0
5 1 1 2 2 3 3 4 4 5 5 6
0 5 0 5 0Time5(min) 0 5 0 5 0
Which of the following statements are correct? (CE-2017)
(i) The elevator never moves directly from any non-ground floor to another non-ground floor over the
one hour period
(ii) The elevator stays on the fourth floor for the longest duration over the one hour period
(a) Only i (b) Only ii
(c) Both i and ii (d) Neither i nor ii

KEY for Data Interpretation


01. (d) 02. (c) 03. (a) 04. (b) 05. (d) 06. (a) 07. (d) 08. (a) 09. (a) 10. (d)
11. (b) 12. (b) 13. (d) 14.(c) 15. (c) 16. (b) 17. (b) 18. (a) 19. (d) 20. (c)
21. (c) 22. (d) 23. (a) 24. (b) 25. (b) 26. (b) 27. (c) 28. (a) 29. (120) 30. (d)
31. (d) 32. (a) 33. (c) 34. (b) 35. (d) 36. (d) 37. (a) 38. (c) 39. (b) 40. (c)
41. (c) 42. (a) 43. (b) 44. (c) 45. (d)
78 Numerical Ability

You might also like